You are on page 1of 68

1

DR. ARVIND’S BIOLOGY CLASSES


(A Unit of Med-Xel Tutorials)

Awake, Arise and Stop not


till the Goal is reached.
CONTENTS

S. NO. TOPIC PAGE NO.


CLASS 12th
REPRODUCITON
1. Reproduction in Organisms NIL
2. Sexual Reproduction in Flowering Plants 2-6
3. Human Reproduction 7-13
4. Reproductive Health 14-16
GENETICS & EVOLUTION
5. Principles of Inheritance and Variation 17-23
6. Molecular Basis of Inheritance 24-29
7. Evolution 30-33
BIOLOGY IN HUMAN WELFARE
8. Human Health & Diseases 34-39
9. Strategies for Enhancement in Food 40-42
Production
10. Microbes in Human Welfare 43-46

S. NO. TOPIC PAGE NO.


BIOTECHNOLOGY
11. Biotechnology 47-52
ECOLOGY
12. Organisms and Populations 53-56
13. Ecosystem 57-61
14. Biodiversity and Conservation 62-64
15. Environmental Issues 65-68
2
DR. ARVIND’S BIOLOGY CLASSES
(A Unit of Med-Xel Tutorials)

REPRODUCTION 7. In which one of the following pollination is


autogamous?
SEXUAL REPRODUCTION IN FLOWERING a) Geitonogamy b) Xenogamy
PLANTS c) Chasmogamy d) Cleistogamy
Ans. (d)
CBSE PRELIMS – 2009 8. What would be the number of chromosomes
of the aleurone cells of a plant with 42
chromosomes in its root tip cells?
1. Vegetative propagation in mint occurs by:
a) 42 b) 63 c) 84 d) 21
a) Rhizome b) Sucker
Ans. (b)
c) Runner d) Offset
9. Wind pollination is common in:
Ans. (b)
a) Legumes b) Lilies
c) Grasses d) Orchids
CBSE PRELIMS – 2010 Ans. (c)
Hint: Wind pollination - Grasses; corn
2. Apomictic embryos in Citrus arise from: Most of the land plants use insects or
a) Diploid egg wind for pollination.
b) Synergids The wind and water pollinated flowers are
c) Maternal sporophytic tissue in ovule not very colourful and do not produce
d) Antipodal cells nectar. Lilies and orchids are colourful
Ans. (c) flowers, so are mostly insect pollinated.
3. Wind pollinated flowers are: Legumes are self pollinated usually.
a) Small, producing nectar and dry pollen
b) Small, brightly coloured, producing large
number of pollen grains CBSE PRELIMS – 2012
c) Small, producing large number of dry
pollen grains
10. Even in absence of pollinating agents seed-
d) Large, producing abundant nectar and
setting is assured in
pollen
a) Zostera b) Salvia
Ans. (c)
c) Fig d) Commellina
4. Transfer of pollen grains from the anther to
Ans. (d)
the stigma of another flower of the same
11. An organic substance that can withstand
plant is called:
environmental extremes and cannot be
a) Autogamy b) Xenogamy
degraded by any enzyme is
c) Geitonogamy d) Karyogamy
a) Sporopollenin b) Lignin
Ans. (c)
c) Cellulose d) Cuticle
Ans. (a)
CBSE PRELIMS – 2011 12. Both autogamy and geitonogamy are prevented in:
a) Cucumber b) Castor
c) Maize d) Papaya
5. Filiform apparatus is a characteristic feature Ans. (d)
of: Hint:
a) Suspensor b) Egg - Cucumber has male and female flowers
c) Synergid d) Zygote both on the same plant.
Ans. (c) - Castor has male and female flowers
6. Nucellar polyembryony is reported in separate but on same plant
species of: - Maize has male and female inflorescences
a) Citrus b) Gossypium (spikelets) borne separately but on same plant
c) Triticum d) Brassica - Papaya is dioecious plant. So both autogamy
Ans. (a) and geitonogamy are prevented in it.
3
DR. ARVIND’S BIOLOGY CLASSES
(A Unit of Med-Xel Tutorials)
CBSE MAINS – 2010 CBSE MAINS – 2012

13. Examine the figures (A – D) given below and 17. Plants with ovaries having only one or a few
select the right option out of 1 – 4, in which ovules, are generally pollinated by:
all the four structures A, B, C and D are a) Birds b) Wind
identified correctly c) Bees d) Butterflies
Ans. (b)
Structures
18. Which one of the following statements is wrong?
a) Pollen grains in some plants remain
viable for months
b) Intine is made up of cellulose and pectin
c) When pollen is shed at two-celled state,
double fertilization does not take place
d) Vegetative cell is larger than generative cell
Ans. (c)
19. What is the function of germ pore?
a) Initiation of pollen tube
b) Release of male gametes
c) Emergence of radicle
d) Absorption of water for seed germination
Ans. (a)

NEET – 2013
A B C D
a) Rhizome Sporangiophore Polar cell Globule 20. Meiosis takes place in:
b) Runner Archegoniophore Synergid Antheridium
a) Gemmule b) Megaspore
c) Meiocyte d) Conidia
c) Offset Antheridiophore Antipodals Oogonium
Ans. (c)
d) Sucker Seta Megaspore Gemma cup 21. Perisperm differs from endosperm in :
mother
cell a) Being a diploid tissue
Ans. (c) b) Its formation by fusion of secondary
14. Vegetative propagation is Pistia occurs by nucleus with several sperms
a) Stolen b) Offset c) Being a haploid tissue
c) Runner d) Sucker d) Having no reserve food
Ans. (b) Ans. (a)
22. Monoecious plant of Chara shows occurrence
of:
CBSE MAINS – 2011 a) Upper antheridium and lower oogonium
on the same plant
15. In angiosperms, functional megaspore
b) Upper oogonium and lower antheridium
develops into:
on the same plant
a) Embryo sac b) Ovule
c) Antheridiophore and archegoniophore on
c) Endosperm d) Pollen sac
the same plant
Ans. (a)
d) Stamen and carpel on the same plant
16. What is common between vegetative
Ans. (b)
reproduction and Apomixis?
23. Which one of the following statements is
a) Both are applicable to only dicot plants
correct?
b) Both bypass the flowering phase
a) Endothecium produces the microspores
c) Both occur round the year
b) Tapetum nourishes the developing pollen
d) Both produces progeny identical to the parent
c) Hard outer layer of pollen is called intine
Ans. (d)
d) Sporogenous tissue is haploid
Ans. (b)
4
DR. ARVIND’S BIOLOGY CLASSES
(A Unit of Med-Xel Tutorials)
AIPMT – 2014 • Apogamy – Formation of sporophyte
from gametophyte without involving the
24. Geitonogamy involves:
formation and fusion of gametes.
a) Fertilization of a flower by the pollen from
• Cleistogamy – the flowers are intersexual
a flower of another plant belonging to a
distant population. and remain closed causing self-
b) Fertilization of a flower by the pollen from pollination, they do not need pollinators.
another flower of the same plant. • Geitonogamy – the pollen grains of one
c) Fertilization of a flower by the pollen from flower are transferred to stigma of
the same flower. another flower belonging to either same
d) Fertilization of a flower by the pollen from • plant or genetically similar plant. This is
a flower of another plant in the same similar genetically to autogamy which is
population self-pollination.
Ans. (b) 30. In ginger vegetative propagation occurs
25. Function of filiform apparatus is to through:
a) Guide the entry of pollen tube a) Offsets b) Bulbils
b) Recognize the suitable pollen at stigma c) Runners d) Rhizome
c) Stimulate division of generative cell Ans. (d)
d) Produce nectar 31. Which one of the following statements is not
Ans. (a) true?
26. Pollen tablets are available in the market for: a) Pollen grains of some plants cause
a) Ex situ conservation severe allergies and bronchial afflictions
b) In vitro fertilization in some people
c) Breeding programmes b) The flowers pollinated by flies and bats
d) Supplementing food secrete foul odour to attract them
Ans. (d) c) Honey is made by bees by digesting
27. Which one of the following is wrong about pollen collected from flowers
Chara? d) Pollen grains are rich in nutrients and
a) Globule is male reproductive structure they are used in the form of tablets and
b) Upper oogonium and lower round syrups
antheridium Ans. (c)
c) Globule and nucule present on the same plant Hint: Option (c) is totally wrong as honey is
d) Upper antheridium and lower oogonium made by bees from nectar collected from
Ans. (d) flowers.
Option (b) is to be taken as correct as
some authors say that flowers pollinated
AIPMT – 2015 by bats have fruity odour and some
others say they may have a disagreeable
28. Transmission tissue is characteristic feature of: odour like that of sour milk.
a) Solid style b) Dry stigma 32. The hilum is a scar on the
c) Wet stigma d) Hollow style a) Fruit, where it was attached to pedicel
Ans. (a) b) Fruit, where style was present
29. Which one of the following may require c) Seed, where micropyle was present
pollinators, but is genetically similar to d) Seed, where funicle was attached
autogamy? Ans. (d)
a) Xenogamy b) Apogamy 33. Which of the following are the important
c) Cleistogamy d) Geitonogamy floral rewards to the animal pollinators?
Ans. (d) a) Nectar and pollen grains
Hint: Xenogamy - Cross pollination between b) Floral fragrance and calcium crystals
two flowers of genetically different c) Protein pellicle and stigmatic exudates
flowers. d) Colour and large size of flower
Ans. (a)
5
DR. ARVIND’S BIOLOGY CLASSES
(A Unit of Med-Xel Tutorials)
AIPMT RETEST – 2015 41. Which of the following statements is not correct?
a) Insects that consume pollen or nectar
34. The wheat grain has an embryo with one without bringing about pollination are
large, shield-shaped cotyledon known as: called pollen/nectar robbers
a) Epiblast b) Coleorrhiza b) Pollen germination and pollen tube
c) Scutellum d) Coleoptile growth are regulated by chemical
Ans. (c) components of pollen interacting with
35. Filiform apparatus is characteristic feature of: those of the pistil
a) Generative cell b) Nucellar embryo c) Some reptiles have also been reported as
c) Aleurone cell d) Synergids pollinators in some plant species
Ans. (d) d) Pollen grains of many species can
36. In angiosperms, microsporogenesis and germinate on the stigma of a flower, but
megasporogenesis: only one pollen tube of the same species
a) Occur in anther grows into the style
b) Form gametes without further divisions Ans. (d)
c) Involve meiosis 42. Seed formation without fertilization in
d) Occur in ovule flowering plants involves the process of:
Ans. (c) a) Budding
37. Which of the following pairs is not correctly b) Somatic hybridisation
c) Apomixis
matched?
d) Sporulation
Mode of reproduction Example Ans. (c)
a) Offset Water hyacinth 43. Which one of the following statements is not true?
b) Rhizome Banana a) Exine of pollen grains is made up of
c) Binary fission Sargassum sporopollenin
d) Conidia Penicillium b) Pollen grains of many species cause
Ans. (c) severe allergies
38. Coconut water from a tender coconut is: c) Stored pollen in liquid nitrogen can be
used in the crop breeding programmes
a) Immature embryo
d) Tapetum helps in the dehiscence of
b) Free nuclear endosperm anther
c) Innermost layers of the seed coat Ans. (d)
d) Degenerated nucellus
NEET-2; 2016
Ans. (b)
39. Male gametophyte in angiosperms produces: 44. In majority of angiosperms
a) Two sperms and a vegetative cell a) Egg has a filiform apparatus
b) Single sperm and a vegetative cell b) There are numerous antipodal cells
c) Reduction division occurs in the
c) Single sperm and two vegetative cells megaspore mother cells
d) Three sperms d) A small central cell is present in the
Ans. (a) embryo sac
Ans. (c)
AIPMT – 2016 Hint: In angiosperms the synergids have
40. The coconut water from tender coconut filiform apparatus, but in some even the
represents: egg cell has filiform apparatus. There are 3
a) Fleshy mesocarp antipodal cells. The central cell is large in
b) Free nuclear proembryo size which has two polar nucleii.
c) Free nuclear endosperm 45. Pollination in water hyacinth and water lily is
d) Endocarp brought about by the agency of
Ans. (c) a) Water b) Insects or wind
c) Birds d) Bats
Ans. (b)
6
DR. ARVIND’S BIOLOGY CLASSES
(A Unit of Med-Xel Tutorials)
46. The ovule of an angiosperm is technically
equivalent to
a) Megasporangium
b) Megasporophyll
c) Megaspore mother cell
d) Megaspore
Ans. (a)
47. Study the four statements (A-D) given below
and select the two correct ones out of them:
(i) Definition of biological species was
given by Ernst Mayr.
(ii) Photoperiod does not affect
reproduction in plants.
(iii) Binomial nomenclature system was
given by R.H. Whittaker
(iv) In unicellular organisms, reproduction is
synonymous with growth.
The two correct statements are
a) (ii) and (iii) b) (iii) and (iv)
c) (i) and (iv) d) (i) and (ii)
Ans. (c)

NEET- 2017
50. Flowers which have single ovule in the ovary
and are packed into inflorescence are
usually pollinated by:
a) Water b) Bee
c) Wind d) Bat
Ans. (c)
51. Functional megaspore in an angiosperm
develops into:
a) Ovule b) Endosperm
c) Embryo sac d) Embryo
Ans. (c)
52. Double fertilization is exhibited by:
a) Gymnosperms b) Algae
c) Fungi d) Angiosperms
Ans. (d)
53. A dioecious flowering plant prevents both:
a) Autogamy and xenogamy
b) Autogamy and geitonogamy
c) Geitonogamy and xenogamy
d) Cleistogamy and xenogamy
Ans. (b)
54. Attractants and rewards are required for
a) Anemophily b) Entomophily
c) Hydrophily d) Cleistogamy
Ans. (b)
7
DR. ARVIND’S BIOLOGY CLASSES
(A Unit of Med-Xel Tutorials)

HUMAN REPRODUCTION 5. Given below is a diagrammatic sketch of a


portion of human male reproductive system.
CBSE PRELIMS – 2009 Select the correct set of the names of the
parts labelled A, B, C, D:
1. Which one of the following is the correct
matching of the events occurring during
menstrual cycle?
(a) Development Secretory phase and
of corpus increased secretion of
luteum progesterone.
(b) Menstruation Breakdown of myometrium
and ovum not fertilized.
(c) Ovulation LH and FSH attain peak
A B C D
level and sharp fall in the a) Vas Seminal Bulboure- Prostate
secretion of progesterone. deferens vesicle thrall gland
(d) Proliferative Rapid regeneration of b) Ureter Seminal Prostate Bulboure-
phase myometrium and maturation vesicle thral gland
of Graafian follicle. c) Ureter Prostate Seminal Bulboure-
Ans. (a) vesicle thral gland
2. The correct sequence of spermatogenetic
d) Vas Seminal Prostate Bulboure-
stages leading to the formation of sperms in
a mature human testis is: deferens vesicle thrall gland
a) Spermatid-spermatocyte-spermatogonia- Ans. (d)
sperms 6. A change in the amount of yolk and its
b) Spermatogonia-spermatid-spermatocyte- distribution in the egg will affect:
sperms a) Number of blastomeres produced
c) Spermatocyte-spermatogonia-spermatid- b) Fertilization
sperms c) Formation of zygote
d) Spermatogonia-spermatocyte-spermatid- d) Pattern of cleavage
sperms Ans. (d)
Ans. (d) 7. Seminal plasma in humans is rich in:
3. Foetal ejection reflex in human female is a) Glucose and certain enzymes but has no
induced by: calcium
a) Fully developed foetus and placenta b) Fructose and certain enzymes but poor
b) Differentiation of mammary glands in calcium
c) Pressure exerted by amniotic fluid
c) Fructose, calcium and certain enzymes
d) Release of oxytocin from pituitary
Ans. (a) d) Fructose and calcium but has no
4. Which one of the following is the most likely enzymes
Ans. (c)
root cause why menstruation is not taking
Hint: Seminal plasma is the term used for
place in regularly cycling human female? semen. Calcium is present in prostatic
a) Maintenance of high concentration of secretion.
sex-hormones in the blood stream
b) Retention of well-developed corpus luteum
c) Fertilisation of the ovum
d) Maintenance of the hypertrophical
endometrial lining
Ans. (c)
8
DR. ARVIND’S BIOLOGY CLASSES
(A Unit of Med-Xel Tutorials)
CBSE PRELIMS – 2010 14. The signals for parturition originate from:
a) Fully developed foetus only
8. Vasa efferentia are the ductules leading from: b) Placenta only
a) Epididymis to urethra c) Placenta as well as fully developed foetus
b) Testicular lobules to rete testis d) Oxytocin released from maternal pituitary
c) Rete testis to vas deferens Ans. (c)
d) Vas deferens to epididymis 15. The second maturation division of the
Ans. (c) mammalian ovum occurs:
Hint: The path followed by sperms is a) In the Graafian follicle following the first
seminiferous tubules  tubuli recti  rete maturation division
testis  Vasa efferentia  epididmis  b) Shortly after ovulation before the ovum
vas deferens  ejaculatory duct  makes entry into the Fallopian tube
urethra. None of the above options is c) Until after the ovum has been
correct. The best option seems to be (c). penetrated by a sperm
9. The first movements of the foetus and d) Until the nucleus of the sperm has fused
appearance of hair on its head are usually with that of the ovum
observed during which month of pregnancy? Ans. (c)
a) Third month b) Fourth month 16. Which one of the following statements about
c) Fifth month d) Sixth month morula in humans is correct?
Ans. (c) a) It has more cytoplasm and more DNA
10. Which one of the following statements about than an uncleaved zygote
human sperm is correct? b) It has almost equal quantity of cytoplasm
a) Acrosome serves no particular function as an uncleaved zygote but much more
b) Acrosome has a conical pointed structure DNA
used for piercing and penetrating the egg c) It has far less cytoplasm as well as less
resulting in fertilization DNA than in an uncleaved zygote
c) The sperm lysins in the acrosome dissolve d) It has more or less equal quantity of
the egg envelope facilitating fertilization cytoplasm and DNA as in uncleaved zygote
d) Acrosome serves as a sensory structure Ans. (b)
leading the sperm towards the ovum
Ans. (c)
11. Seminal plasma in human males is rich in: CBSE PRELIMS – 2011
a) Ribose and potassium 17. If for some reason, the vasa efferentia in the
b) Fructose and calcium human reproductive system get blocked, the
c) Glucose and calcium gametes will not be transported from:
d) DNA and testosterone a) Testes to epididymis
Ans. (b) b) Epididymis to vas deferens
Hint: Fructose is mainly present in secretions of c) Ovary to uterus
seminal vesicles and calcium in prostatic d) Vagina to uterus
secretions. Ans. (a)
12. Sertoli cells are found in: 18. The testes in humans are situated outside
a) Pancreas and secrete cholecystokinin the abdominal cavity inside a pouch called
b) Ovaries and secrete progesterone scrotum. The purpose served is for:
c) Adrenal cortex and secrete adrenaline a) Maintaining the scrotal temperature
d) Seminiferous tubules and provide lower than the internal body temperature
nutrition to germ cells b) Escaping any possible compression by
Ans. (d) the visceral organs
13. The part of Fallopian tube closest to the ovary is: c) Providing more space for the growth of
a) Ampulla b) Isthmus epididymis
c) Infundibulum d) Cervix d) Providing a secondary sexual feature for
Ans. (c) exhibiting the male sex
Ans. (a)
9
DR. ARVIND’S BIOLOGY CLASSES
(A Unit of Med-Xel Tutorials)
19. The figure given below depicts a a) Survival of sperm depends on the pH of
diagrammatic sectional view of the female the medium and is more active in
reproductive system of humans. Which one alkaline medium
set of three parts out of I-VI have been b) Viability of sperm is determined by its motility
correctly identified? c) Sperms must be concentrated in a thick
suspension
d) Sperm is viable for only up to 24 hours
Ans. (d)

CBSE MAINS – 2010

24. Signals from fully developed foetus and


placenta ultimately lead to parturition which
a) (II) Endometrium, (III) Infundibulum, (IV) requires the release of
Fimbriae a) Estrogen from placenta
b) (III) Infundibulum, (IV) Fimbriae, (V) Cervix b) Oxytocin from maternal pituitary
c) (IV) Oviducal funnel, (V) Uterus, (VI) Cervix c) Oxytocin from foetal pituitary
d) (I) Perimetrium, (II) Myometrium, (III) d) Relaxin from placenta
Fallopian tube Ans. (b)
Ans. (b) 25. In human female the blastocyst
a) Forms placenta even before implantation
CBSE PRELIMS – 2012 b) Gets implanted into uterus 3 days after
ovulation
20. In a normal pregnant woman, the amount of c) Gets nutrition from uterine endometrial
total gonadotropin activity was assessed. secretion only after implantation
The result expected was: d) Gets implanted in endometrium by the
a) High level of circulating HCG to stimulate trophoblast cells
endometrial thickening Ans. (d)
b) High levels of FSH and LH in uterus to 26. Secretions from which one of the following are
stimulate endometrial thickening rich in fructose, calcium and some enzymes?
c) High level of circulating HCG to stimulate a) Male accessory glands
estrogen and progesterone synthesis b) Liver
d) High level of circulating FSH and LH in c) Pancreas
the uterus to stimulate implantation of the d) Salivary glands
embryo Ans. (a)
Ans. (c)
21. Signals for parturition originate from CBSE MAINS – 2011
a) Oxytocin released from maternal pituitary
b) Placenta only 27. What happens during fertilisation in humans
c) Fully developed foetus only after many sperms reach close to the ovum?
d) Both placenta as well as fully developed a) Secretions of acrosome helps one sperm enter
foetus cytoplasm of ovum through zona pellucida
Ans. (d) b) All sperms except the one nearest to the
22. The Leydig cells as found in the human body ovum lose their tails
are the secretory source of: c) Cells of corona radiata trap all the sperms
a) Intestinal mucus b) Glucagon except one
c) Androgens d) Progesterone d) Only two sperms nearest the ovum
Ans. (c) penetrate zona pellucida
23. Which one of the following statements is false Ans. (a)
in respect of viability of mammalian sperm?
10
DR. ARVIND’S BIOLOGY CLASSES
(A Unit of Med-Xel Tutorials)
28. About which day in a normal human NEET – 2013
menstrual cycle does rapid secretion of LH
(popularly called LH-surge) normally occurs? 31. What is the correct sequence of sperm
a) 14th day b) 20th day formation?
c) 5th day d) 11th day a) Spermatogonia, spermatozoa, spermatocyte,
spermatid
Ans. (a)
b) Spermatogonia, spermatocyte, spermatid,
spermatozoa
CBSE MAINS – 2012 c) Spermatid, spermatocyte, spermatogonia.
spermatozoa
d) Spermatogonia, spermatocyte, spermatozoa,
29. The secretory phase in the human menstrual spermatid
cycle is also called: Ans. (b)
a) Luteal phase and lasts for about 13 days 32. Which one of the following is not the function
b) Follicular phase and lasts for about 13 days of placenta? It:
c) Luteal phase and lasts for about 6 days a) Facilitates removal of carbon dioxide and
waste material from embryo.
d) Follicular phase and lasting for about 6 days
b) Secretes oxytocin during parturition.
Ans. (a)
c) Facilitates supply of oxygen and
30. Identify the human development stage nutrients to embryo.
shown below as well as the related right place d) Secretes estrogen.
of its occurrence in a normal pregnant Ans. (b)
woman, and select the right option for the two 33. Product of sexual reproduction generally
together. generates:
a) New genetic combination leading to
variation
b) Large biomass
c) Longer viability of seeds
d) Prolonged dormancy
Ans. (a)
34. Menstrual flow occurs due to lack of:
a) Oxytocin b) Vasopressin
c) Progesterone d) FSH
Options:
Ans. (c)
Developmental stage Site of occurrence
a) Blastocyst Uterine wall
AIPMT – 2014
b) 8-celled morula Starting point of Fallopian
tube 35. The shared terminal duct of the reproductive
c) Late morula Middle part of Fallopian and urinary system in the human male is:
tube a) Vasa efferentia b) Urethra
d) Blastula End part of Fallopian tube c) Ureter d) Vas deferens
Ans. (a) Ans. (b)
Hint: The embryo with 8-16 blastomeres is 36. The main function of mammalian corpus
called a morula, and is present in the end luteum is to produce:
part of fallopian tube. Morula forms the a) relaxin only
b) estrogen only
blastocyst as it reaches the uterine
c) progesterone
cavity and then blastocyst implants in
d) human chorionic gonadotropin
the endometrium. Thus, the options (b), Ans. (c)
(c), (d) are all wrong.
11
DR. ARVIND’S BIOLOGY CLASSES
(A Unit of Med-Xel Tutorials)
37. Select the correct option describing
gonadotropin activity in a normal pregnant AIPMT RETEST – 2015
female
a) High level of hCG stimulates the 42. Ectopic pregnancies are referred to as:
thickening of endometrium a) Pregnancies with genetic abnormality
b) High level of FSH and LH stimulates the b) Implantation of embryo at site other than
thickening of endometrium uterus
c) High level of FSH and LH facilitate
c) Implantation of defective embryo in the
implantation of the embryo
d) High level of hCG stimulates the synthesis uterus
of estrogen and progesterone d) Pregnancies terminated due to hormonal
Ans. (d) imbalance
Hint: In a normal pregnancy – Ans. (b)
• hCG from the placenta maintains the 43. Which of the following events is not
corpus luteum and the secretion of associated with ovulation in human female?
estrogen and progesterone by it. a) Decrease in estradiol
• The FSH and LH levels are suppressed b) Full development of Graafian follicle
during pregnancy thus both option (b) c) Release of secondary oocyte
and (c) are incorrect. d) LH surge
Ans. (a)
AIPMT – 2015 44. Which of the following layers in an antral
follicle is acellular?
38. Which of these is not an important component a) Granulosa b) Theca interna
of initiation of parturition in humans? c) Stroma d) Zona pellucida
a) Synthesis of prostaglandins Ans. (d)
45. In human females, meiosis-II is not
b) Release of oxytocin
c) Release of prolactin completed until?
d) Increase in estrogen and progesterone ratio a) puberty b) fertilization
c) uterine implantation d) birth
Ans. (c)
Ans. (b)
39. Capacitation refers to changes in the:
a) Ovum before fertilization AIPMT – 2016
b) Ovum after fertilization 46. Fertilization in humans is practically feasible
c) Sperm after fertilization only if:
d) Sperm before fertilization a) the ovum and sperms are transported
Ans. (d) simultaneously to Ampullary – isthmic
40. Hysterectomy is surgical removal of: junction of the fallopian tube
a) Prostate gland b) Vas-deference b) the ovum and sperms are transported
c) Mammary glands d) Uterus simultaneously to ampullary –isthmic
Ans. (d) junction of the cervix
41. Which of the following cells during c) the sperms are transported into cervix
gametogenesis is normally diploid? within 48 hrs of release of ovum in uterus
a) Spermatid d) the sperms are transported into vagina just
b) Spermatogonia after the release of ovum in fallopian tube
c) Secondary polar body Ans. (a)
d) Primary polar body Hint: Fertilization in humans is possible in
Ans. (b) both the case of option (a) and (d). But,
in the question statement the word ‘only’
is written so (a) becomes the better
answer.
12
DR. ARVIND’S BIOLOGY CLASSES
(A Unit of Med-Xel Tutorials)
47. Changes in GnRH pulse frequency in 51. Match Column – I with Column – II and select
females is controlled by circulating levels of: the correct option using the codes given
a) Estrogen and inhibin below:
b) Progesterone only Column – I Column – II
c) Progesterone and inhibin 1. Mons pubis (i) Embryo formation
d) Estrogen and progesterone 2. Antrum (ii) Sperm
Ans. (d) 3. Trophectoderm (iii) Female external
48. Identify the correct statement on ‘inhibin’: genitalia
4. Nebenkern (iv) Graafian follicle
a) Is produced by granulose cells in ovary
and inhibits the secretion of FSH. Codes
b) Is produced by granulose cells in ovary 1 2 3 4
and inhibits the secretion of LH. a) (iii) (iv) (ii) (i)
c) Is produced by nurse cells in testes and b) (iii) (iv) (i) (ii)
inhibits the secretion of LH. c) (iii) (i) (iv) (ii)
d) Inhibits the secretion of LH, FSH and d) (i) (iv) (iii) (ii)
Prolactin. Ans. (b)
Ans. (a) Hint: Nebenkern is a helical structure of the
Hint: Inhibin (hormone) is secreted by proximal tail region of spermatozoaqn of
granulosa cells of the ovaries in female some insects derived from mitochondria.
and Sertoli cells of testes in males. It 52. Several hormones like hCG, hPL, estrogen,
acts primarily to inhibit the secretion of progesterone are produced by
FSH from the anterior pituitary gland.
a) Ovary b) Placenta
49. Select the incorrect statement:
c) Fallopian tube d) Pituitary
a) LH triggers ovulation in ovary.
Ans. (b)
b) LH and FSH decrease gradually during
the follicular phase.
c) LH triggers secretion of androgens from NEET – 2017
the Leydig cells.
d) FSH stimulates the Sertoli cells which 53. Select the correct route for the passage of
help in spermiogenesis sperms in male frogs:
a) Testes  Bidder’s canal  Kidney 
Ans. (b)
Vasa efferentia  Urinogenital duct 
Cloaca
NEET 2; 2016 b) Testes  Vasa efferentia  Kidney 
Seminal Vesicle  Urinogenital duct 
Cloaca
50. Which of the following depicts the correct c) Testes  Vasa efferentia  Bidder’s
pathway of transport of sperms? canal  Ureter  Cloaca
a) Rete testis  Efferent ductules  d) Testes  Vasa efferentia  Kidney 
Epididymis  Vas deferens. Bidder’s canal  Urinogenital duct 
b) Rete testis  Epididymis  Efferent Cloaca
Ans. (d)
ductules  Vas deferens
54. A temporary endocrine gland in the human
c) Rete testis  Vas deferens  Efferent
body is:
ductules  Epididymis
a) Pineal gland b) Corpus
d) Efferent ductules  Rete testis  Vas
cardiacum
deferens  Epididymis
c) Corpus luteum d) Corpus allatum
Ans. (a) Ans. (c)
13
DR. ARVIND’S BIOLOGY CLASSES
(A Unit of Med-Xel Tutorials)
55. GnRH, a hypothalamic hormone, needed in
reproduction, acts on:
a) anterior pituitary gland and stimulates
secretion of LH and oxytocin.
b) anterior pituitary gland and stimulates
secretion of LH and FSH
c) posterior pituitary gland and stimulates
secretion of oxytocin and FSH
d) posterior pituitary gland and stimulates
secretion of LH and relaxin.
Ans. (b)
56. Capacitation occurs in:
a) Rete testis
b) Epididymis
c) Vas deferens
d) Female Reproductive tract
Ans. (d)
14
DR. ARVIND’S BIOLOGY CLASSES
(A Unit of Med-Xel Tutorials)
REPRODUCTIVE HEALTH 7. What is the figure given below showing in
particular?
CBSE PRELIMS – 2010
1. Cu ions released from copper-releasing
Intra Uterine Devices (IUDs):
a) Prevent ovulation
b) Make uterus unsuitable for implantation
c) Increase phagocytosis of sperms
d) Suppress sperm motility
Ans. (d) a) Uterine cancer b) Tubectomy
2. In vitro fertilization is a technique that c) Vasectomy d) Ovarian cancer
involves transfer of which one of the Ans. (b)
following into the fallopian tube?
a) Zygote only CBSE MAINS – 2011
b) Embryo only, upto 8 cell stage
c) Either zygote or early embryo upto 8 cell 8. The technique called gamete intrafallopian
stage transfer (GIFT) is recommended for those
d) Embryo of 32 cell stage females:
Ans. (c) a) Who cannot produce an ovum
3. The permissible use of the technique b) Who cannot retain the foetus inside uterus
amniocentesis is for: c) Whose cervical canal is too narrow to
a) Detecting any genetic abnormality allow passage for the sperms
b) Detecting sex of the unborn foetus d) Who cannot provide suitable environment
c) Artificial insemination for fertilisation
d) Transfer of embryo into the uterus of a Ans. (a)
surrogate mother
Ans. (a) NEET – 2013
CBSE PRELIMS – 2011
9. Artificial insemination means:
4. Which one of the following is the most a) artificial introduction of sperms of a
widely accepted method of contraception in healthy donor into the vagina
India, as at present? b) introduction of sperms of a healthy donor
a) Cervical caps directly into the ovary.
b) Tubectomy c) transfer of sperms of a healthy donor to a
c) Diaphragms test tube containing ova
d) IUDs’ (Intra uterine devices) d) transfer of sperms of husband to a test
Ans. (d) tube containing ova
5. Medical Termination of Pregnancy (MTP) is Ans. (a)
considered safe up to how many weeks of 10. Which of the following cannot be detected in
pregnancy? a developing foetus by amniocentesis?
a) Eight weeks b) Twelve weeks a) Down syndrome
c) Eighteen weeks d) Six weeks b) Jaundice
Ans. (b) c) Klinefelter syndrome
d) Sex of the foetus
CBSE PRELIMS – 2012 Ans. (b)
6. The Test-tube Baby Programme employs 11. One of the legal methods of birth control is:
which one of the following techniques? a) by having coitus at the time of day break
a) Intra uterine insemination (IUI) b) by a premature ejaculation during coitus
b) Gamete intra fallopian transfer (GIFT) c) abortion by taking an appropriate medicine
c) Zygote intra fallopian transfer (ZIFT) d) by abstaining from coitus from day 10 to
d) Intra cytoplasmic sperm injection (ICSI) 17 of the menstrual cycle
Ans. (c) Ans. (d)
15
DR. ARVIND’S BIOLOGY CLASSES
(A Unit of Med-Xel Tutorials)
Hint: Both the options (c) and (d) are correct.
If it was asked which is the method for AIPMT – 2016
prevention of pregnancy / conception 16. Which of the following approaches does not
then abstinence would have been the give the defined action of contraceptive?
only correct option. a) Intra uterine Increase phagocytosis
devices of sperms, suppress
sperm motility and
AIPMT – 2014 fertilizing capacity of
sperms
12. Assisted reproductive technology, IVF b) Hormonal Prevent/retard entry of
involves transfer of: contraceptives sperms, prevent
a) Embryo with 16 blastomeres into the ovulation & fertilization
fallopian tube c) Vasectomy Prevents
b) Ovum into the fallopian tube. spermatogenesis
c) Zygote into the fallopian tube d) Barrier Prevent fertilization
d) Zygote into the uterus. methods
Ans. (c)
Hint: In IVF and embryo transfer; the zygote Ans. (c)
or early embryo (upto 8 blastomeres) 17. In context of Amniocentesis, which of the
could be transferred into fallopian tube following statement is incorrect?
(ZIFT) and embryos with more than 8 a) It is used for prenatal sex determination
blastomeres into the uterus ( IUT). b) It can be used for detection of Down
13. Tubectomy is a method of sterilization in syndrome
which c) It can be used for detection of Cleft palate
a) Uterus is removed surgically d) It is usually done when a woman is
b) Small part of the fallopian tube is removed between 14-16 weeks pregnant
or tied up. Ans. (c)
c) Ovaries are removed surgically
d) Small part of vas deferens is removed or tied NEET 2; 2016
up
Ans. (b) 18. Which of the following is hormone releasing
14. Which of the following is a hormone IUD?
releasing intra Uterine Device (IUD)? a) LNG-20 b) Multiload 375
a) Vault b) Multiload 375 c) Lippes loop d) Cu7
c) LNG-20 d) Cervical cap Ans. (a)
Ans. (c) 19. Which of the following is incorrect regarding
vasectomy?
AIPMT RETEST – 2015 a) No sperm occurs in seminal fluid
b) No sperm occurs in epididymis
15. A childless couple can be assisted to have a c) Vasa deferentia is cut and tied
child through a technique called GIFT. The d) Irreversible sterility
full form of this technique is: Ans. (b)
a) Gamete inseminated fallopian transfer 20. Embryo with more than 16 blastomeres
b) Gamete intra fallopian transfer formed due to in vitro fertilization is
c) Gamete internal fertilization and transfer transferred into
d) Germ cell internal fallopian transfer a) Uterus b) Fallopian tube
c) Fimbriae d) Cervix
Ans. (b)
Ans. (a)
16
DR. ARVIND’S BIOLOGY CLASSES
(A Unit of Med-Xel Tutorials)
NEET – 2017 mentioned. For low sperm count both
Artificial insemination and ICSI can be
21. Match the following sexually transmitted used depending on the degree of
diseases (Column – I) with their causative oligospermia. For ‘very low sperm count’
agent (Column – II) and select the correct ICSI seems to be the better option.
option.
Column – I Column – II
A. Gonorrhea (i) HIV
B. Syphilis (ii) Neisseria
C. Genital Warts (iii) Treponema
D. AIDS (iv) Human Papilloma -
Virus
Options:
A B C D
a) (ii) (iii) (iv) (i)
b) (iii) (iv) (i) (ii)
c) (iv) (ii) (iii) (i)
d) (iv) (iii) (ii) (i)
Ans. (a)
22. The function of copper ions in copper
releasing IUD’s is:
a) They suppress sperm motility and
fertilising capacity of sperms.
b) They inhibit gametogenesis
c) They make uterus unsuitable for
implantation
d) They inhibit ovulation
Ans. (a) as per CBSE key.
Hint: Both options (a) and (c) are correct.
Copper ions supress the sperm
motility and fertilizing capacity of sperms
and are spermicidal in nature. However,
in addition it increases the inflammatory
reaction in the uterus and makes the
uterus unsuitable for implantation, the
reason why it is also useful as an
emergency contraceptive.
23. In case of a couple where the male is
having a very low sperm count, which
technique will be suitable for fertilization?
a) Intrauterine transfer
b) Gamete intracytoplasmic fallopian
transfer
c) Artificial Insemination
d) Intracytoplasmic sperm injection
Ans. (c) as per CBSE key.
Hint: In this question it is mentioned”Very
low sperm count”, which is a qualitative
criteria. No numerical value to define
degree of oligospermia has been
17
DR. ARVIND’S BIOLOGY CLASSES
(A Unit of Med-Xel Tutorials)

GENETICS & EVOLUTION d) Galactosemia is an inborn error of


metabolism
PRINCIPLES OF INHERITANCE AND VARIATIONS Ans. (b)
Hint: Baldness is a sex-influenced trait. It
CBSE PRELIMS – 2009 behaves as autosomal dominant in males
and autosomal recessive in females.
1. Sickle cell anaemia is: 5. Alzhimer disease in humans is associated
a) Caused by a change in a single base with the deficiency of:
pair of DNA a) Acetylcholine
b) Characterized by elongated sickle like b) Gamma aminobutyric acid (GABA)
RBCs with a nucleus c) Dopamine
c) An autosomal linked dominant trait d) Glutamic acid
d) Caused by substitution of valine by Ans. (a)
glutamic acid in the beta globin chain of
haemoglobin CBSE PRELIMS – 2010
Ans. (a)
2. The most popularly known blood grouping is 6. Which one of the following cannot be
the ABO grouping. It is named ABO and not explained on the basis of Mendel’s Law of
ABC, because “O” in it refers to having: Dominance?
a) One antibody only – either anti-A or anti- a) Factors occur in pairs
B on the RBCs b) The discrete unit controlling a particular
b) No antigens A and B on RBCs character is called a factor
c) Other antigens besides A and B on RBCs c) Out of one pair of factors one is
d) Overdominance of this type on the dominant, and the other is recessive.
genes for A and B types d) Alleles do not show any blending and
Ans. (b) both the characters recover as such in F2
3. Study the pedigree chart given below: generation
Ans. (d)
7. The genotype of a plant showing the
dominant phenotype can be determined by:
a) Back cross b) Test cross
c) Dihybrid cross d) Pedigree analysis
Ans. (b)
8. ABO blood groups in humans are controlled
What does it show? by the gene I. It has three alleles – IA, IB and
a) The pedigree chart is wrong as this is not i. Since there are three different alleles, six
possible. different genotypes are possible. How many
b) Inheritance of a recessive sex-linked phenotypes can occur?
disease like haemophilia a) Two b) Three c) One d) Four
c) Inheritance of a sex-linked inborn error of Ans. (d)
metabolism like phenylketonuria 9. Select the correct statement from the ones
d) Inheritance of a condition like phenylketonuria given below with respect to dihybrid cross
as an autosomal recessive trait a) Tightly linked genes on the same chromo-
Ans. (d) some show very few recom-binations.
4. Select the incorrect statement from the b) Tightly linked genes on the same
following: chromosome show higher recombinations
a) Small population size results in random c) Genes far apart on the same chromosome
genetic drift in a population show very few recombinations
b) Baldness is a sex-limited trait d) Genes loosely linked on the same
c) Linkage is an exception to the principle chromosome show similar recom-
of independent assortment in heredity binations as the tightly linked ones
Ans. (a)
18
DR. ARVIND’S BIOLOGY CLASSES
(A Unit of Med-Xel Tutorials)
10. Which one of the following symbols and its 15. F2 generation in a Mendelian cross showed
representation, used in human pedigree that both genotypic and phenotypic ratios are
analysis is correct? same as 1 : 2 : 1. It represents a case of:
a) = male affected a) Dihybrid cross
b) Monohybrid cross with complete dominance
b) = mating between relatives c) Monohybrid cross with incomplete dominance
d) Co-dominance
c) = unaffected male Ans. (c)
Hint: In both incomplete dominance and co-
d) = unaffected female dominance, the genotypic and phenotypic
Ans. (b) ratios are 1:2:1. Probably option (c) is a
better answer, since monohybrid cross is
mentioned with it.
CBSE PRELIMS – 2011
11. Which one of the following conditions CBSE MAINS – 2010
correctly describes the manner of
16. Which one of the following statements about
determining the sex in the given example?
the particular entity is true?
a) Homozygous sex chromosomes (ZZ)
a) Centromere is found in animal cells,
determine female sex in Birds
which produces aster during cell division
b) XO type of sex chromosomes determine
b) The gene for producing insulin is present
male sex in grasshopper
in every body cell
c) XO condition in humans as found in Turner
Syndrome, determines female sex c) Nucleosome is formed of nucleotides
d) Homozygous sex chromosomes (XX) d) DNA consists of a core of eight histones
produce male in Drosophila Ans. (b)
Ans. (b) 17. Study the pedigree chart of a certain family
12. What are those structures that appear as given below and select the correct conclusion
‘beads’ – on – string’ in the chromosomes which can be drawn for the character
when viewed under electron microscope?
a) Genes b) Nucleotides
c) Nucleosomes d) Base pairs
Ans. (c)
13. When two unrelated individuals or lines are a) The female parent is heterozygous
crossed, the performance of F1 hybrid is b) The parents could not have had a normal
often superior to both its parents. This daughter for this character
phenomenon is called: c) The trait under study could not be colour
a) Heterosis b) Transformation blindness
c) Splicing d) Metamorphosis d) The male parent is homozygous dominant
Ans. (a) Ans. (a)
Hint: The option (c) is also correct i.e., the trait
under study could not be color blindness.
CBSE PRELIMS – 2012 As per pedigree mother does not seem to
be a carrier of sex linked recessive
14. A normal-visioned man whose father was disorder as the symbol for carrier of sex
colour-blind, marries a woman whose father linked recessive disorder [] has not
was also colour-blind. They have their first been used. If mother is not a carrier of
child as a daughter. What are the chances sex linked recessive disorder, then none
that this child would be colour-blind? of the daughters can be a sufferer. But
a) zero percent b) 25% still (a) is a better choice.
c) 50% d) 100%
Ans. (a)
19
DR. ARVIND’S BIOLOGY CLASSES
(A Unit of Med-Xel Tutorials)
18. In Antirrhinum two plants with pink flowers 24. Represented below is the inheritance pattern
were hybridized. The F1 plants produced red, of a certain type of traits in humans. Which
pink and white flowers in the proportion of 1 one of the following conditions could be an
red, 2 pink and 1 white. What could be the example of this pattern?
genotype of the two plants used for
hybridization? Red flower colour is
determined by RR, and white by rr genes.
a) rrrr b) RR c) Rr d) rr
Ans. (c)
19. A cross in which an organism showing a
dominant phenotype in crossed with the
recessive parent in order to know its
genotype is called: a) Haemophilia b) Thalassemia
a) Monohybrid cross b) Back cross c) Phenylketonuria d) Sickle cell anaemia
c) Test cross d) Dihybrid cross Ans. (a)
Ans. (c) 25. A test cross is carried out to:
20. ABO blood grouping is controlled by gene I a) Assess the number of alleles of a gene
which has three alleles and show co- b) Determine whether two species or
dominance. There are six genotypes. How varieties will breed successfully
many phenotypes in all are possible? c) Determine the genotype of a plant at F2
a) Six b) Three c) Four d) Five d) Predict whether two traits are linked
Ans. (c) Ans. (c)
21. The fruit fly Drosophila melanogaster was found
to be very suitable for experimental verification of NEET – 2013
chromosomal theory of inheritance by Morgan
and his colleagues because: 26. If both parents are carriers for thalessemia,
a) It reproduces parthenogenetically which is an autosomal recessive disorder,
b) A single mating produces two young flies what are the chances of pregnancy resulting
c) Smaller female is easily recognizable in an affected child?
from larger male a) 25% b) 100%
d) It completes life cycle in about two weeks c) no chance d) 50%
Ans. (d) Ans. (a)
27. Which Mendelian idea is depicted by a cross
in which the F1 generation resembles both
CBSE MAINS – 2011 the parents?
22. Test cross in plants or in Drosophila involves a) inheritance of one gene
crossing: b) co-dominance
a) Between two genotypes with recessive trait c) incomplete dominance
b) Between two F1 hybrids d) law of dominance
c) The F1 hybrid with a double recessive genotype Ans. (b)
d) Between two genotypes with dominant trait 28. The incorrect statement with regard to
Ans. (c) Haemophilia is:
23. Which one of the following conditions of the a) It is a dominant disease
zygotic cell would lead to the birth of a normal b) A single protein involved in the clotting of
human female child? blood is affected
a) two X chromosomes c) It is a sex-linked disease
b) Only one Y chromosome d) It is a recessive disease
c) Only one X chromosome Ans. (a)
d) One X and one Y chromosome
Ans. (a)
20
DR. ARVIND’S BIOLOGY CLASSES
(A Unit of Med-Xel Tutorials)
29. Which of the following statements is not true
of two genes that show 50% recombination AIPMT – 2015
frequency?
a) The genes show independent 34. How many pairs of contrasting characters in
assortment pea plants were studied by Mendel in his
b) If the genes are present on the same experiments?
chromosome, they undergo more than a) Six b) Eight
one crossovers in every meiosis c) Seven d) Five
c) The genes may be on different chromosomes Ans. (c)
d) The genes are tightly linked 35. The movement of a gene from one linkage
Ans. (d) group to another is called:
Hint: If two genes show a 50% recombination a) Duplication b) Translocation
frequency it means that they are not linked c) Crossing over d) Inversion
and show independent assortment. They may Ans. (b)
be on the same chromosome or are more
likely to be on different chromosomes. 36. Multiple alleles are present:
If they are on the same chromosome then a) At different loci on the same chromosome
the distance between then is large i.e., b) At the same locus of the chromosome
nearly 50cm. We know that larger the c) On non-sister chromatids
distance between genes on the same d) On different chromosomes
chromosome greater are the chances of Ans. (b)
more than one cross overs in meiosis. 37. Alleles are:
a) True breeding homozygotes
AIPMT – 2014 b) Different molecular forms of a gene
30. A man whose father was colour blind c) Heterozygotes
married a woman who had a colour blind d) Different phenotype
mother and normal father. What percentage Ans. (b)
of male children of the couple will be colour 38. An abnormal human baby with ‘XXX’ sex
blind? chromosomes was born due to:
a) 75% b) 25% c) 0% d) 50% a) Formation of abnormal ova in the mother
Ans. (d) b) Fusion of two ova and one sperm
31. In a population of 1000 individuals 360 c) Fusion of two sperms and one ovum
belong to genotype AA 480 to Aa and the d) Formation of abnormal sperms in the
remaining 160 to aa. Based on this data, father
the frequency of allele A in the population Ans. As per CBSE website the Ans. is (a).
is: Hint: (a and d) In this question both option (a)
a) 0.7 b) 0.4 c) 0.5 d) 0.6 and (d) are correct. XXX condition in
Ans. (d) humans can originate due to Maternal
32. A human female with Turner’s syndrome: defect in meiosis I or meiosis II or due to
a) Is able to produce children with normal paternal defect in meiosis II. These defects
husband can give rise to abnormal egg with XX or
b) Has 45 chromosomes with XO abnormal sperm with XX which if fuses with
c) Has one additional X chromosome normal gamete of opposite sex will give
d) Exhibits male characters rise to an abnormal zygote with XXX.
Ans. (b) Reference – Harrison’s Principles of
33. Fruit colour in squash is an example of: Internal Medicine.
a) Inhibitory genes
b) Recessive epistasis
c) Dominant epistasis
d) Complementary genes
Ans. (c)
21
DR. ARVIND’S BIOLOGY CLASSES
(A Unit of Med-Xel Tutorials)
AIPMT RETEST – 2015 If we consider the grandsons only
through the daughter then the answer will
39. A gene showing codominance has: be 0.5.
a) One allele dominant on the other 43. Balbiani rings are sites of:
b) Alleles tightly linked on the same a) Lipid synthesis
chromosome b) Nucleotide synthesis
c) Alleles that are recessive to each other c) Polysaccharide synthesis
d) Both alleles independently expressed in d) RNA and protein synthesis
the heterozygote Ans. (d)
Ans. (d) Hint: According to Textbooks,” Genetics by
40. The term “linkage” was coined by: Strickberger and “Genetics by PK Gupta”,
a) T.H. Morgan b) T. Boveri Balbiani rings are sites of RNA synthesis.
c) G. Mendel d) W. This RNA then goes to cytoplasm and
Sutton makes proteins at the ribosomes. Balbiani
Ans. (a) rings being a part of chromosomes cannot
41. In the following human pedigree, the filled be a site of protein synthesis. Thus, even
symbols, represent the affected individuals. the option (d) “RNA and protein synthesis”
Identify the type of given pedigree. is not completely correct.
44. In his classic experiments on pea plants,
Mendel did not use:
a) Seed colour b) Pod length
c) Seed shape d) Flower position
Ans. (b)
45. A pleiotropic gene:
a) is expressed only in primitive plants
b) is a gene evolved during Pliocene
c) controls a trait only in combination with
a) Autosomal dominant
another gene
b) X-linked recessive
d) controls multiple traits in an individual
c) Autosomal recessive
Ans. (d)
d) X-linked dominant
Ans. (c)
42. A colour blind man marries a woman with
AIPMT – 2016
normal sight who has no history of colour
blindness in her family. What is the probability 46. In a testcross involving F1 dihybrid flies, more
of their grandson being colour blind? parental-type offspring were produced than the
recombinant-type offspring. This indicates:
a) 0.5 b) 1 c) Nil d) 0.25
a) Chromosomes failed to separate during
Ans. (d) meiosis
Hint: Here it is not specified whether b) The two genes are linked and present on
grandsons are through daughter or son the same chromosome
and thus we have to consider all the c) Both of the characters are controlled by
grandsons i.e., both through the son and more than one gene
the daughter. In such a case the answer d) The two genes are located on two different
chromosomes
comes out to be one out of four (0.25). Ans. (b)
If we consider the grandsons only through
the son then the answer will be Nil.
22
DR. ARVIND’S BIOLOGY CLASSES
(A Unit of Med-Xel Tutorials)
47. Match the terms in Column I with their 51. Pick out the correct statements:
description in Column II and choose the (i) Haemophilia is a sex-linked recessive
correct option: disease.
Column I Column II (ii) Down’s syndrome is due to aneuploidy.
A) Dominance (i) Many genes govern a single (iii) Phenylketonuria is an autosomal
character recessive gene disorder
b) (ii) In a heterozygoes organism only (iv) Sickle cell anaemia is an X-linked
Codominance one allele expresses itself
recessive gene disorder
c) Pleiotropy (iii) In a heterozygous
a) (ii) & (iv) are correct
organism both alleles express
themselves fully. b) (i), (iii) and (iv) are correct
d) Polygenic (iv) A single gene influences many c) (i), (ii) and (iii) are correct
inheritance characters d) (i) and (iv) are correct
Code: (A) (B) (C) (D) Ans. (c)
a) (ii) (iii) (iv) (i)
b) (iv) (i) (ii) (iii)
c) (iv) (iii) (i) (ii) NEET 2; 2016
d) (ii) (i) (iv) (iii)
Ans. (a) 52. Which one of the following generates new
48. A cell at telophase stage is observed by a genetic combinations leading to variation?
student in a plant brought from the field. He a) Vegetative reproduction
tells his teacher that this cell is not like other b) Parthenogenesis
cells at telophase stage. There is no formation c) Sexual reproduction
of cell plate and thus the cell is containing d) Nucellar polyembryony
more number of chromosomes as compared Ans. (c)
to other dividing cells. This would result in: 53. The mechanism that causes a gene to move
a) Polyploidy from one linkage group to another is called
b) Somaclonal variation a) Inversion b) Duplication
c) Polyteny c) Translocation d) Crossing over
d) Aneuploidy Ans. (c)
Ans. (a) 54. A true breeding plant is
49. Which of the following most appropriately a) One that is able to breed on its own
describes haemophilia? b) Produced due to cross-pollination among
a) X-linked recessive gene disorder unrelated plants
b) Chromosomal disorder c) Near homozygous and produces
c) Dominant gene disorder offspring of its own kind
d) Recessive gene disorder d) Always homozygous recessive in its
Ans. (a) genetic constitution
50. A tall true breeding garden pea plant is Ans. (c)
crossed with a dwarf true breeding garden 55. If a colour-blind man marries a woman who
pea plant. When the F1 plants were selfed is homozygous for normal colour vision, the
the resulting genotypes were in the ratio of: probability of their son being colour blind is
a) 1 : 2 : 1 :: Tall heterozygous : Tall a) 0 b) 0.5 c) 0.75 d) 1
homozygous : dwarf Ans. (a)
b) 3 : 1 :: Tall : Dwarf 56. In Hardy-Weinberg equation, the frequency
c) 3 : 1 :: Dwarf : Tall of heterozygous individual is represented by
d) 1 : 2 : 1 :: Tall homozygous : Tall a) p2 b) 2 pq c) pq d) q2
heterozygous : Dwarf Ans. (b)
Ans. (d)
23
DR. ARVIND’S BIOLOGY CLASSES
(A Unit of Med-Xel Tutorials)
NEET - 2017 62. A disease caused by an autosomal primary
non-disjunction is;
a) Down’s Syndome
57. Which one from those given below is the
period for Mendel’s hybridization b) Klinefelter’s Syndrome
experiments? c) Turner’s Syndrome
a) 1856 – 1863 b) 1840 – 1850 d) Sickle Cell Anemia
c) 1857 – 1869 d) 1870 – 1877 Ans. (a)
Ans. (a)
58. Among the following characters, which one was
not considered by Mendel in his experiments on
pea?
a) Stem – Tall or Dwarf
b) Trichomes – Glandular or non-glandular
c) Seed – Green or Yellow
d) Pod – Inflated or Constricted
Ans. (b)
59. The genotypes of a Husband and Wife are
IAIB and IAi.
Among the blood types of their children, how
many different genotypes and phenotypes
are possible?
a) 3 genotypes ; 3 phenotypes
b) 3 genotypes ; 4 phenotypes
c) 4 genotypes ; 3 phenotypes
d) 4 genotypes ; 4 phenotypes
Ans. (c)
60. The association of histone H1 with a
nucleosome indicates:
a) Transcription is occurring.
b) DNA replication is occurring
c) The DNA is condensed into a Chromatin
Fibre.
d) The DNA double helix is exposed
Ans. (c)
61. Thalassemia and sickle cell anemia are
caused due to a problem in globin molecule
synthesis. Select the correct statement.
a) Both are due to a qualitative defect in
globin chain synthesis.
b) Both are due to a quantitative defect in
globin chain synthesis.
c) Thalassemia is due to less synthesis of
globin molecules
d) Sickle cell anemia is due to a quantitative
problem of globin molecules.
Ans. (c)
24
DR. ARVIND’S BIOLOGY CLASSES
(A Unit of Med-Xel Tutorials)
MOLECULAR BASIS OF INHERITANCE 8. Infectious proteins are present in:
a) Satellite viruses b) Gemini viruses
CBSE PRELIMS – 2009 c) Prions d) Viroids
Ans. (c)
1. What is not true for genetic code? 9. The one aspect which is not a salient feature
a) It is degenerate of genetic code, is its being:
b) It is unambiguous a) Specific b) Degenerate
c) A codon in mRNA is read in a non- c) Ambiguous d) Universal
contiguous fashion Ans. (c)
d) It is nearly universal 10. Satellite DNA is useful tool in
Ans. (c) a) Genetic engineering
2. Point mutation involves: b) Organ transplantation
a) Duplication c) Sex determination
b) Deletion d) Forensic science
c) Insertion Ans. (d)
d) Change in single base pair 11. Which one of the following does not follow
Ans. (d) the central dogma of molecular biology?
3. Semiconservative replication of DNA was a) HIV b) Pea
first demonstrated in: c) Mucor d) Chlamydomonas
a) Streptococcus pneumoniae Ans. (a)
b) Salmonella typhimurium 12. Select the two correct statements out of the
c) Drosophila melanogaster four given below about lac operon.
d) Escherichia coli (i) Glucose or galactose may bind with the
Ans. (d) repressor and inactivate it.
4. T.O. Diener discovered a: (ii) In the absence of lactose the repressor
a) Infectious protein binds with the operator region
b) Bacteriophage (iii) The z-gene codes for permease
c) Free infectious RNA (iv) This was elucidated by Francois Jacob
d) Free infectious DNA and Jacque Monod
Ans. (c) The correct statements are:
Hint: These are viriods which are smallest a) (i) and (ii) b) (ii) and (iii)
known agents of infection c) (i) and (iii) d) (ii) and (iv)
5. Removal of introns and joining the exons in a Ans. (d)
defined order in a transcription unit is called: Hint: z-gene codes for beta galactosidase,
a) Transformation b) Capping y-gene codes for permease and a-gene
c) Splicing d) Tailing codes for transacetylase. Lactose
Ans. (c) binds with repressor and inactivates it.
6. Whose experiments cracked the DNA and
discovered unequivocally that a genetic code
is a “triplet”? CBSE PRELIMS – 2011
a) Morgan and Sturtevant
b) Beadle and Tatum 13. Which one of the following also acts as a
c) Nirenberg and Mathaei catalyst in a bacterial cell?
d) Hershey and Chase a) 5 sr RNA b) sn RNA
Ans. (c) c) hn RNA d) 23 sr RNA
Ans. (d)
Hint: In prokaryotes 23S rRNA and in
CBSE PRELIMS – 2010 eukaryotes 28S rRNA acts as peptidyl
7. Virus envelope is known as transferase enzyme.
a) Core b) Capsid
c) Virion d) Nucleoprotein
Ans. (b)
25
DR. ARVIND’S BIOLOGY CLASSES
(A Unit of Med-Xel Tutorials)
CBSE PRELIMS – 2012 20. Ribosomal RNA is actively synthesized in:
a) Nucleolus b) Nucleoplasm
14. If one strand of DNA has the nitrogenous c) Ribosomes d) Lysosomes
base sequence as ATCTG, what would be Ans. (a)
the complementary RNA strand sequence? 21. Removal of introns and joining of exons in a
a) UAGAC b) AACTG defined order during transcription is called:
c) ATCGU d) TTAGU a) Inducing b) Slicing
Ans. (a) c) Splicing d) Looping
15. PCR and Restriction Fragment Length Ans. (c)
Polymorphism are the methods for: 22. Which statement is wrong for viruses?
a) Genetic transformation a) All of them have helical symmetry
b) DNA sequencing b) They have ability to synthesize nucleic
c) Genetic Fingerprinting acids and proteins
d) Study of enzymes c) Antibiotics have no effect on them
Ans. (c) d) All are parasites
16. Given below is the diagrammatic Ans. (a)
representation of one of the categories of 23. Given below is the diagram of a
small molecular weight organic compounds in bacteriophage. In which one of the options
the living tissues. Identify the category shown all the four parts A, B, C and D are correct?
and the one blank component “X” in it.

Category Component
a) Amino acid NH2
b) Nucleotide Adenine
A B C D
c) Nucleoside Uracil
a) Tail fibres Head Sheath Collar
d) Cholesterol Guanin b) Sheath Collar Head Tail fibres
Ans. (c) c) Head Sheath Collar Tail fibres
17. Removal of RNA polymerase III from d) Collar Tail Head Sheath
fibres
nucleoplasm will affect the synthesis of:
a) hn RNA b) m RNA Ans. (c)
c) r RNA d) t RNA 24. The lac operon consists of
Ans. (d) a. Four regulatory genes only
18. A single strand of nucleic acid tagged with a b. One regulatory gene and three structural
radioactive molecule is called: genes
a) Selectable marker b) Plasmid c. Two regulatory genes & two structural genes
c) Probe d) Vector d. Three regulatory genes and three
Ans. (c) structural genes
19. Which one of the following is not a part of a Ans. (b)
transcription unit in DNA? 25. In eukaryotic cell transcription, RNA splicing
A terminator b) A promoter and RNA capping take place inside the
c) The structural gene d) The inducer a) Ribosomes b) Nucleus
Ans. (d) c) Dictyosomes d) ER
Ans. (b)
26
DR. ARVIND’S BIOLOGY CLASSES
(A Unit of Med-Xel Tutorials)
26. The 3’-5’ phosphodiester linkages inside a Hint: (A) and (D) are true
polynucleotide chain serve to join ▪ (A) mentioned in NCERT as a statement.
a) One DNA strand with the other DNA strand Ideally it should have been Adenine or
b) One nucleoside with another nucleoside Deoxyadenosine.
c) One nucleotide with another nucleotide ▪ (B) False; Regulation of lac operon by
d) One nitrogenous base with pentose sugar repressor is a negative regulation.
Ans. (c) ▪ (C) False; Human genome has 30,000
genes.
CBSE MAINS – 2011 31. Which one of the following is a wrong
statement regarding mutations?
27. The unequivocal proof of DNA as the genetic
a) UV and Gamma rays are mutagens
material came from the studies on a:
b) Change in a single base pair of DNA
a) Bacterium b) Fungus
does not cause mutation
c) Viroid d) Bacterial virus
c) Deletion and insertion of base pairs
Ans. (d)
cause frame-shift mutation
Hint: The unequivocal proof came from
d) Cancer cells commonly show chromosomal
experiment of Hershey and Chase on
aberrations
bacteriophages.
Ans. (b)
32. In which of the following reproduction of
CBSE MAINS – 2012 bacteria phage is essential?
a) Conjugation b) Transformation
28. Which one of the following represents a c) Transduction d) All above
palindromic sequence in DNA? Ans. (c)
a) 5'–CATTAG–3' b) 5'–GATACC–3'
3'–GATAAC–5' 3'–CCTAAG–5' NEET – 2013
c) 5'–GAATTC–3' d) 5'–CCAATG–3'
3'–CTTAAG–5' 3'–GAATCC–5' 33. Which enzyme/s will be produced in a cell in
Ans. (c) which there is a nonsense mutation in the lac
Y gene?
29. What is it that forms the basis of DNA
a. Transacetylase
Fingerprinting? b. Lactose permease and
a) The relative amount of DNA in the ridges transacetylase
and grooves of the fingerprints c.  - galactosidase
b) Satellite DNA occurring as highly d. Lactose permease
repeated short DNA segments Ans. (c)
c) The relative proportions of purines and Hint: In the lac – operon; there are three
pyrimidines in DNA structural genes – Z; ‘y’ and ‘a’. They are
d) The relative difference in the DNA translated in sequence (i.e., first Z then y
occurrence in blood, skin and saliva then a). If there is a non sense mutation in
Ans. (b) the ‘y’ gene, then only Z will get translated
30. Read the following four statements (A-D): and y and a will not get translated.
(A) In transcription, adenosine pairs with uracil Therefore only the enzyme coded by Z (i.e.,
(B) Regulation of lac operon by repressor is  galactosidase) will be produced. The
referred to as positive regulation other two enzymes (i.e., ‘y’ coding for)
(C) The human genome has approximately Permease and (‘a’ coding for)
50,000 genes transacetylase will not be produced.
(D) Haemophilia is a sex-linked recessive
disease
How many of the above statements are right?
a) Four b) One c) Two d) Three
Ans. (c)
27
DR. ARVIND’S BIOLOGY CLASSES
(A Unit of Med-Xel Tutorials)
34. The diagram shows an important concept in Transcription is the general term for
the genetic implication of DNA. Fill in the synthesis of any kind of RNA on a DNA
blanks A to C. template.
A B Proposed by
NCERT TB Class XII itself on pg 109, 110
DNA mRNA Protein and 111 mentions.
C • There is a single DNA-dependent RNA
a. A – transcription B – translation C – polymerase that catalyses transcription
Francis Crick of all types of RNA in bacteria.
b. A – translation B – extension C – • The RNA polymerase 1 transcribes
Rosalind Franklin rRNAs and the RNA polymerase III is
c. A – transcription B – replication C responsible for transcription of tRNA, 5
–James Watson srRNA and sn RNA
d. A- translation B – transcription C Thus, none of the given options is wrongly
– Erevin Chargaff matched.
Ans. (a) 38. An analysis of chromosomal DNA using
Southern hybridization technique does not
AIPMT – 2014 use
35. Select the correct option: a) PCR b) Electrophoresis
c) Blotting d) Autoradiography
Direction of RNA Direction of reading Ans. (a)
synthesis of the template DNA 39. Transformation was discovered by:
strand a) Watson and Crick
a) 3’ ---- 5’ 3’ ---- 5’ b) Meselson and Stahl
b) 5’ ---- 3’ 3’ ---- 5’ c) Hershey and Chase
c) 3’ ---- 5’ 5’ ---- 3’ d) Griffith
d) 5’ ---- 3’ 5’ ---- 3’ Ans. (d)
40. Commonly used vectors for human genome
Ans. (b) sequencing are:
36. Which of the following shows coiled RNA a) T / A Cloning Vectors
strand and capsomeres? b) T – DNA
a) Retrovirus b) Polio virus c) BAC and YAC
c) Tobacco mosaic virus d) Measles virus d) Expression Vectors
Ans. (c) Ans. (c)
Hint: TMV and Measles virus both have 41. Viruses have
coiled RNA strand and capsomeres. Thus a) Both DNA and RNA
options (c) and (d) are both correct. b) DNA enclosed in a protein coat
Ref.; Ananthanarayan TB of Microbiology c) Prokaryotic nucleus
and Harrison’s Principles of Internal d) Single chromosome
Medicine. Ans. (b)
37. Which one of the following is wrongly
matched? AIPMT – 2015
a) Operon – Structural genes, operator and
42. In sea urchin DNA, which is double stranded,
promoter
b) Transcription – Writing information from 17% of the bases were shown to be cytosine.
DNA to t-RNA The percentages of the other three bases
c) Translation – Using information in m- expected to be present in this DNA are:
RNA to make protein a) G17%, A 16.5%, T 32.5%
d) Repressor protein – Binds to operator to b) G 17%, A 33%, T 33%
stop enzyme synthesis. c) G 8.5%, A 50%, T 24.5%
Ans. (b)
d) G 34%, A 24.5%, T 24.5%
None of these is wrongly matched.
Hint: Biology by Campbell and Reece – Ans. (b)
28
DR. ARVIND’S BIOLOGY CLASSES
(A Unit of Med-Xel Tutorials)
43. Gene regulation governing lactose operon of 47. Satellite DNA is important because it:
E. coli that involves the lac I gene product is: a) codes for proteins needed in cell cycle
a) negative and inducible because b) shows high degree of polymorphism in
repressor protein prevents transcription population and also the same degree of
b) negative and repressible because polymorphism in an individual, which is
repressor protein prevents transcription heritable from parents to children
c) feedback inhibition because excess of - c) does not code for proteins and is same in
galactosidase can switch off transcription all members of the population
d) positive and inducible because it can be d) codes for enzymes needed for DNA
induced by lactose replication
Ans. (a) Ans. (b)
Hint: In lactose operon of E. coli, the lac I
gene product is the repressor which AIPMT – 2016
blocks the operator gene so that the
structural genes are unable to form 48. Which of the following statements is wrong for
mRNAs. Thus, regulation of lac operon viroids?
by repressor is referred to as negative a) They are smaller than viruses
regulation. Due to the repressor the lac b) They cause infections
operon remains in inactive state and is c) Their RNA is of high molecular weight
thus inducible. d) They lack a protein coat
Ans. (c)
49. Which of the following statements is not true
AIPMT RETEST – 2015 for cancer cells in relation to mutations?
44. Select the wrong statement: a) Mutations destroy telomerase inhibitor
a) The viroids were discovered by D.J. b) Mutation inactivate the cell control
Ivanowski c) Mutations inhibit production of telomerase
b) W.M. Stanley showed that viruses could d) Mutations in proto-oncogenes accelerate
be crystallized the cell cycle
c) The term ‘contagium vivum fluidum’ was Ans. (c)
coined by M.W. Beijerinek Hint: Telomerase replaces short bits of DNA
d) Mosaic disease in tobacco and AIDS in (telomeres) which are shortened when
human being are caused by viruses cell undergoes mitosis. Telomerase
Ans. (a) enzyme is high in cells which divide
Hint: Viroids were discovered by T.O. Diener. repeatedly e.g., stem cells. The enzyme
45. Which one of the following is not applicable to is at a very low level or absent in somatic
cells. Persistent high telomerase level in
RNA? the cells will allow them to undergo
a) Complementary base pairing unbounded growth as is the feature of
b) 5’ phosphoryl and 3’ hydroxyl ends cancerous cells.
c) Heterocyclic nitrogenous bases 50. Which one of the following is the starter
d) Chargaff’s rule codon?
Ans. (d) a) UGA b) UAA
c) UAG d) AUG
46. Identify the correct order of organisation of
Ans. (d)
genetic material from largest to smallest: 51. Which of the following is required as
a) Chromosome, gene, genome, nucleotide inducer(s) for the expression of Lac operon?
b) Genome, chromosome, nucleotide, gene a) galactose b) lactose
c) Genome, chromosome, gene, nucleotide c) lactose and galactose d) glucose
d) Chromosome, genome, nucleotide, gene Ans. (b)
Ans. (c)
29
DR. ARVIND’S BIOLOGY CLASSES
(A Unit of Med-Xel Tutorials)
52. Which of the following is not required for any 58. DNA-dependent RNA polymerase catalyzes
of the techniques of DNA fingerprinting transcription on one strand of the DNA which
available at present? is called the
a) Zinc finger analysis a) Template strand b) Coding strand
b) Restriction enzymes c) Alpha strand d) Antistrand
c) DNA-DNA hybridization Ans. (a)
d) Polymerase chain reaction
Ans. (a) NEET – 2017
Hint: Zinc finger proteins have many functions
like DNA recognition, RNA packaging, 59. If there are 999 bases in an RNA that codes
targeting specific genomic sites, for a protein with 333 amino acids, and the
transcriptional studies; but is a recent base at position 901 is deleted such that the
discovery and under research. Not yet length of the RNA becomes 998 bases, how
widely used. many codons will be altered?
a) 1 b) 11 c) 33 d) 333
NEET 2; 2016 Ans. (c)
53. A non-proteinaceous enzyme is 60. DNA fragments are:
a) Lysozyme b) Ribozyme a) Positively charged
c) Ligase d) Deoxyribonuclease b) Negatively charged
Ans. (b) c) Neutral
54. Taylor conducted the experiments to prove d) Either positively or negatively charged
semiconservative mode of chromosome depending on their size
replication on Ans. (b)
a) Vinca rosea 61. Viroids differ from viruses in having:
b) Vicia faba a) DNA molecules with protein coat
c) Drosophila melanogaster b) DNA molecules without protein coat
d) E. coli c) RNA molecules with protein coat
Ans. (b) d) RNA molecules without protein coat
55. The equivalent of a structural gene is Ans. (d)
a) Muton b) Cistron 62. During DNA replication, Okazaki fragments
c) Operon d) Recon are used to elongate:
Ans. (b) a) The leading strand towards replication
56. Which of the following rRNAs acts as fork.
structural RNA as well as ribozyme in b) The lagging strand towards replication
bacteria? fork.
a) 5 S rRNA b) 18 S rRNA c) The leading strand away from replication
c) 23 S rRNA d) 5.8 S rRNA fork.
Ans. (c) d) The lagging strand away from the
57. A molecule that can act as a genetic material replication fork.
must fulfill the traits given below, except Ans. (d)
a) It should be able to express itself in the 63. Spliceosomes are not found in cells of:
form of ‘Mendelian characters’ a) Plants b) Fungi
b) It should be able to generate its replica c) Animals d) Bacteria
c) It should be unstable structurally and Ans. (d)
chemically 64. The final proof for DNA as the genetic
d) It should provide the scope for slow material came from the experiments of:
changes that are required for evolution a) Griffith
Ans. (c) b) Hershey and Chase
c) Avery, Mcleod and McCarty
d) Hargobind Khorana
Ans. (b)
30
DR. ARVIND’S BIOLOGY CLASSES
(A Unit of Med-Xel Tutorials)

EVOLUTION 5. Which one of the following options gives


one correct example each of convergent
CBSE PRELIMS – 2009 evolution and divergent evolution?
Convergent evolution Divergent
evolution
1. In the case of peppered moth (Biston betularia) a) Thorns of Bougainvillia and Wings of butterflies
the black-coloured form became dominant tendrils of Cucurbita and birds
over the light-coloured form in England during b) Bones of forelimbs of Wings of butterfly
industrial revolution. This is an example of: vertebrates and birds
a) Protective mimicry c) Thorns of Bougainvillia and Eyes of Octopus
b) Inheritance of darker colour character tendrils of Cucurbita and mammals
acquired due to the darker environment d) Eyes of octopus and Bones of forelimbs
c) Natural selection whereby the darker mammals of vertebrates
forms were selected Ans. (d)
d) Appearance of the darker coloured 6. The extinct human who lived 1,00,000 to
individuals due to very poor sunlight 40,000 years ago, in Europe, Asia and parts
Ans. (c) of Africa, with short stature, heavy eye
brows, retreating foreheads, large jaws with
heavy teeth, stocky bodies, a lumbering gait
CBSE PRELIMS – 2010 and stooped posture was:
a) Neanderthal human
2. Darwin’s finches are a good example of: b) Cro-magnan humans
a) Convergent evolution c) Ramapithecus
b) Industrial melanism d) Homo habilis
c) Connecting link Ans. (a)
d) Adaptive radiation 7. Evolution of different species in a given
Ans. (d) area starting from a point and spreading to
other geographical areas is known as:
CBSE PRELIMS – 2011 a) Natural selection
b) Migration
c) Divergent evolution
3. What was the most significant trend in the d) Adaptive radiation
evolution of modern man (Homo sapiens) Ans. (d)
from his ancestors?
a) Upright posture CBSE MAINS – 2010
b) Shortening of jaws
c) Binocular vision 8. Given below are four statements (A-D) each
d) Increasing brain capacity with one or two blanks. Select the option which
Ans. (d) correctly fill up the blanks in two statements
Statements:
CBSE PRELIMS – 2012 (A) Wings of butterfly and birds look alike
and are the results of __(i)_, evolution.
4. What was the most significant trend in the (B) Miller showed that CH4, H2, NH3 and
evolution of modern man (Homo sapiens) (i)_,when exposed to electric discharge
from his ancestors? in a flask resulted in formation of
a) Binocular vision (ii)___.
b) Increasing cranial capacity
c) Upright posture (C) Vermiform appendix is a (i)__ organ
d) Shortening of jaws and an __(ii)__ evidence of evolution.
Ans. (b) (D) According to Darwin evolution took place
due to ___(i)__ and __(ii)__of the fittest.
31
DR. ARVIND’S BIOLOGY CLASSES
(A Unit of Med-Xel Tutorials)
Options: d) Interspecific competition.
a) (D) – (i) Small variations, (ii) Survival, Ans. (a), (c) & (d)
Hint: According to Darwin Organic evolution is
(A) – (i) Convergent due to both interspecific and intraspecific
b) (A) – (i) Convergent, competition.
(B) – (i) Oxygen, (ii) Nucleosides Moreover Competition between closely
related species, means Interspecific
c) (B) – (i) Water vapour, (ii) Amino acids competition. Thus options (a), (c) and (d) are
(C) – (i) Rudimentary, (ii) Anatomical all correct. CBSE has also taken all these
d) (C) – (i) Vestigial, (ii) Anatomical three options as correct options.
(D) – (i) Mutations, (ii) Multiplication 12. The tendency of population to remain in
genetic equilibrium may be disturbed by:
Ans. (a) a) lack of mutations
Hint: Rudimentary means at an early stage of b) lack of random mating
development (Collins) or a body part that c) random mating
is not fully developed. d) lack of migration
9. The most apparent change during the Ans. (b)
evolutionary history of Homo sapiens is 13. Variation in gene frequencies within
traced in populations can occur by chance rather
a) Loss of body hair than by natural selection.
b) Walking upright This is referred to as:
c) Shortening of the jaws a) Random mating b) Genetic load
d) Remarkable increase in the brain size c) Genetic flow d) Genetic drift
Ans. (d) Ans. (d)
14. The process by which organisms with
CBSE MAINS – 2011 (NIL) different evolutionary history evolve similar
phenotypic adaptations in response to a
CBSE MAINS – 2012 common environmental challenge, is called:
a) Non-random evolution
b) Adaptive radiation
10. The idea of mutations was brought forth by:- c) Natural selection
a) Hardy Weinberg, who worked on allele d) Convergent evolution
frequencies in a population Ans. (d)
b) Charles Darwin, who observed a wide 15. The eye of octopus and eye of cat show
variety of organisms during sea voyage different patterns of structure, yet they
c) Hugo de Vries, who worked on evening perform similar function. This is an example
primrose of:
a) Analogous organs that have evolved
d) Gregor Mendel, who worked on Pisum
due to convergent evolution.
sativum
b) Analogous organs that have evolved
Ans. (c)
due to divergent evolution.
c) Homologous organs that have evolved
NEET – 2013 due to convergent evolution.
11. According to Darwin, the organic evolution d) Homologous organs that have evolved
is due to: due to divergent evolution.
a) Competition within closely related Ans. (a)
species.
b) Reduced feeding efficiency in one
species due to the presence of
interfering species.
c) Intraspecific competition
32
DR. ARVIND’S BIOLOGY CLASSES
(A Unit of Med-Xel Tutorials)
AIPMT – 2014 20. A population will not exist in Hardy-Weinberg
equilibrium if:
16. Which one of the following are analogous a) There are no mutations
structures? b) There is no migration
a) Flippers of Dolphin and Legs of Horse c) The population is large
b) Wings of Bat and Wings of Pigeon. d) Individuals mate selectively
c) Gills of Prawn and Lungs of Man.
Ans. (d)
d) Thorns of Bougainvillea & Tendrils of
Cucurbita
Ans. (c) AIPMT RETEST – 2015
Hint: Both (b) and (c) are correct.
Evolution by PK Gupta and Evolution 21. The wings of a bird and the wings of an
by Ridley – Wings of insects, reptiles, birds insect are:
and bats are believed to have developed a) Homologous structures and represent
independently and are analogous since divergent evolution
they perform same function. b) Analogous structures and represent
As for Gills of Prawn and Lungs of convergent evolution
Man, they are structurally different and c) Phylogenetic structures and represent
functionally serve the same function, thus divergent evolution
are analogous. d) Homologous structures and represent
17. Forelimbs of cat, lizard used in walking, convergent evolution
forelimbs of whale used in swimming and Ans. (b)
forelimbs of bats used in flying are an example
of: 22. Industrial melanism is an example of:
a) Convergent evolution a) Neo Darwinism b) Natural selection
b) Analogous organs c) Mutation d) Neo Lamarckism
c) Adaptive radiation Ans. (b)
d) Homologous organs
Ans. (d)
AIPMT – 2016
AIPMT - 2015 23. Which of the following structures is
homologus to the wing of a bird?
18. Which of the following had the smallest a) Wing of a Moth
brain capacity? b) Hind limb of Rabbit
a) Homo sapiens c) Flipper of Whale
b) Homo neanderthalensis d) Dorsal fin of a Shark
c) Homo habilis Ans. (c)
d) Homo erectus 24. Following are the two statements regarding
Ans. (c) the origin of life:
Hint: Brain capacities (i) The earliest organisms that appeared on
• Homo habilis – 650 to 800 cc the earth were non-green and
• Homo erectus – 900 cc presumably anaerobes
• Homo neanderthalensis – 1400 cc (ii) The first autotrophic organisms were the
• Homo sapiens – 1200 – 1600 cc chemoautotrophs that never released
19. Which is the most common mechanism of oxygen
genetic variation in the population of a Of the above statements which one of the
sexually reproducing organism? following options is correct?
a) Chromosomal aberrations a) (ii) is correct but (i) is false
b) Genetic drift b) Both (i) and (ii) are correct
c) Recombination c) Both (i) and (ii) are false
d) Transduction d) (i) is correct but (ii) is false
Ans. (c) Ans. (b)
33
DR. ARVIND’S BIOLOGY CLASSES
(A Unit of Med-Xel Tutorials)
25. Analogous structures are a result of:
a) Convergent evolution NEET – 2017
b) Shared ancestry
c) Stabilizing selection 30. Artificial selection to obtain cows yielding
d) Divergent evolution high milk output represents:
Ans. (a) a) Stabilizing selection as it stabilizes a
character in the population.
b) Directional as it pushes the mean of a
NEET-2; 2016 character in one direction
c) Disruptive as it splits the population into
26. Genetic drift operates in the one yielding higher output and the
a) Small isolated population other lower output
b) Large isolated population d) Stabilizing followed by disruptive as
stabilizes the population to produce high
c) Non-reproductive population
yielding cows.
d) Slow reproductive population Ans. (b)
Ans. (a)
27. The chronological order of human evolution
from early to the recent is
a) Australopithecus  Ramapithecus 
Homo habilis  Homo erectus
b) Ramapithecus  Australopithecus 
Homo habilis  Homo erectus
c) Ramapithecus  Homo habilis 
Australopithecus  Homo erectus
d) Australopithecus  Homo habilis 
Ramapithecus  Homo erectus
Ans. (b)
28. Which of the following is the correct
sequence of events in the origin of life?
I. Formation of protobionts
II. Synthesis of organic monomers
III. Synthesis of organic polymers
IV. Formation of DNA-based genetic
systems
a) I, II, III, IV b) I, III, II, IV
c) II, III, I, IV d) II, III, IV, I
Ans. (c)
34
DR. ARVIND’S BIOLOGY CLASSES
(A Unit of Med-Xel Tutorials)

BIOLOGY IN HUMAN WELFARE


HUMAN HEALTH AND DISEASES CBSE PRELIMS – 2010

CBSE PRELIMS – 2009 7. Consider the following four statements


regarding kindney transplant and select the
1. The letter T in T-lymphocyte refers to: two correct ones out of these.
a) Tonsil b) Thymus (i) Even if a kidney transplant is proper the
c) Thyroid d) Thalamus recipient may need to take immuno-
Ans. (b) suppresants for a long time.
2. Which of the following is a pair of viral diseases? (ii) The cell-mediated immune response is
a) Dysentery, Common Cold responsible for the graft rejection
b) Typhoid, Tuberculosis (iii) The B-lymphocytes are responsible for
c) Ringworm, AIDS rejection of the graft.
d) Common Cold, AIDS (iv) The acceptance or rejection of a kidney
Ans. (d) transplant depends on specific
3. A person likely to develop tetanus is interferons
immunized by administering: The two correct statements are:
a) Wide spectrum antibiotics a) (i) and (ii) b) (ii) and (iii)
b) Weakened germs c) (iii) and (iv) d) (i) and (iii)
c) Dead germs Ans. (a)
d) Preformed antibodies 8. Widal test is used for the diagnosis of:
Ans. (d) a) Typhoid b) Malaria
4. Which one of the following statements is correct? c) Pneumonia d) Tuberculosis
a) Heroin accelerates body functions. Ans. (a)
b) Malignant tumours may exhibit metastasis 9. Ringworm in humans is caused by:
c) Patients who have undergone surgery a) Viruses b) Bacteria
are given cannabinoid to relieve pain. c) Fungi d) Nematodes
d) Benign tumours show the property of Ans. (c)
metastasis. 10. Which one of the following statements is
Ans. (b) correct with respect to AIDS?
5. Globulins contained in human blood plasma a) The causative HIV retrovirus enters helper
are primarily involved in: T-lymphocytes thus reducing their numbers
a) Oxygen transport in the blood b) The HIV can be transmitted through eating
b) Clotting of blood food together with an infected person
c) Defence mechanisms of body c) Drug addicts are least susceptible to HIV
d) Oxmotic balance of body fluids infection
Ans. (c) d) AIDS patients are being fully cured cent
6. Use of anti-histamines and steroids give a per cent with proper care and nutrition
quick relief from: Ans. (a)
a) Cough b) headache 11. Select the correct statement from the ones
c) Allergy d) Nausea given below:
Ans. (c) a) Cocaine is given to patients after surgery
as it stimulates recovery
b) Barbiturates when given to criminals
make them tell the truth
c) Morphine is often given to persons who
have undergone surgery as a pain killer
d) Chewing tobacco lowers blood pressure
and heart rate
Ans. (c)
35
DR. ARVIND’S BIOLOGY CLASSES
(A Unit of Med-Xel Tutorials)
CBSE PRELIMS – 2011 c) Pneumonia pathogen infects alveoli
whereas the common cold affects nose
12. At which stage of HIV infection does one and respiratory passage but not the lungs
usually show symptoms of AIDS? d) Pneumonia is a communicable disease
a) When the infecting retrovirus enters host cells whereas the common cold is a nutritional
b) When viral DNA is produced by reverse deficiency disease
transcriptase Ans. (c)
c) When HIV replicates rapidly in helper 17. In which one of the following options the two
T-lymphocytes and damages large examples are correctly matched with their
number of these particular type of immunity?
d) Within 15 days of sexual contact with an Examples Type of
infected person immunity
Ans. (c) a) Anti-tetanus and anti- Active immunity
13. Which one of the following acts as a snake bite injections
physiological barrier to the entry of b) Saliva in mouth and Physical barriers
microorganisms in human body? Tears in eyes
a) Epithelium of Urogenital tract c) Mucus coating of Physiological
b) Tears epithelium lining the barriers
c) Monocytes urinogenital tract and
d) Skin the HCl in stomach
Ans. (b) d) Polymorphonuclear Cellular barriers
Hint: Epithelium of urogenital tract and skin are leukocytes and
physical barriers. monocytes
Monocytes are cellular barrier. Ans. (d)
Tear, saliva and acid in stomach are Hint: Anti tetanus and anti-snake bite injections –
physiological barriers. passive immunity. Saliva & tears –
14. A certain patient is suspected to be suffering physiological barriers. Mucus coating –
from Acquired Immuno Deficiency physical barrier.
Syndrome. Which diagnostic technique will 18. Cirrhosis of liver is caused by the chronic intake of:
you recommend for its detection? a) Alcohol b) Tobacco (Chewing)
a) ELISA b) MRI c) Cocaine d) Opium
c) Ultra sound d) WIDAL Ans. (a)
Ans. (a) 19. Widal Test is carried out to test:
a) Diabetes mellitus b) HIV/AIDS
CBSE PRELIMS – 2012 c) Typhoid fever d) Malaria
Ans. (c)
15. Which one of the following is not a property
of cancerous cells whereas the remaining CBSE MAINS – 2010
three are? 20. Which one of the following techniques is
a) They do not remain confined in the area safest for the detection of cancers?
of formation a) Magnetic resonance imaging (MRI)
b) They divide in an uncontrolled manner b) Radiography (X- ray)
c) They show contact inhibition c) Computed tomography (CT)
d) They compete with normal cells for vital d) Histopathological studies
nutrients Ans. (a)
Ans. (c) Hint: This question seems to be controversial.
16. Common cold differs from pneumonia in, that: The most definitive and confirmatory
a) Pneumonia can be prevented by a live technique for diagnosis of cancers is
attenuated bacterial vaccine whereas the histopathological examination. MRI
common cold has no effective vaccine. cannot diagnose and differentiate all
b) Pneumonia is caused by a virus while the cancers. Since in the question word
common cold is caused by the bacterium safest has been mentioned, the examiner
Haemophilus influenzae wants option (a) as the correct answer.
36
DR. ARVIND’S BIOLOGY CLASSES
(A Unit of Med-Xel Tutorials)
21. Which one of the following can not be used Ans. (b)
for preparation of vaccines against plague? 24. Common cold is not cured by antibiotics
a) Formalin – inactivated suspensions of because it is:
virulent bacteria a) Caused by a virus
b) Avirulent live bacteria b) Caused by a Gram-positive bacterium
c) Synthetic capsular polysaccharide material c) Caused by a Gram-negative bacterium
d) Heat-killed suspensions of virulent bacteria d) Not an infectious disease
Ans. (c) Ans. (a)
Hint: (This is a tough question and is out of syllabus. 25. Select the correct statement with respect to
Even the books of Microbiology, SPM and diseases and immunisation:
Medicine have not clearly mentioned these a) If due to some reason B-and T-lymphocytes
options. Such questions should better be left are damaged, the body will not produce
unattempted). antibodies against a pathogen.
• The earliest plague vaccine was killed b) Injection of dead/inactivated pathogens
whole cell vaccine but had severe side causes passive immunity
effects so was discontinued. c) Certain protozoans have been used to
• Live plague vaccine was tried mass produce hepatitis B vaccine
experimentally but failed due to severe d) Injection of snake antivenom against snake
reactions. bite is an example of active immunisation
• Killed preparation of virulent strains are Ans. (a)
used as vaccination now. It can be Hint: Using recombinant DNA technology.
killed by heat or formalin. Hepatitis B vaccine has been produced
• Now a days formalin killed preparation from yeast. Injection of snake antivenom
of highly virulent strain 195/P is used which contains preformed antibodies, is an
for vaccination example of passive immunisation. Injection
Capsular polysaccharide vaccines are used in: of dead/inactivated pathogens leads to
(i) Meningococcus vaccine development of active immunity.
(ii) Pneumococcus vaccine
(iii) Parenteral Typhoid vaccine
Thus, the best option is (c). CBSE MAINS – 2012
CBSE MAINS – 2011
26. Read the following four statements (A – D):
22. The pathogen Microsporum responsible for (A) Colostrum is recommended for the new
ringworm disease in humans belongs to the born because it is rich in antigens
same Kingdom of organisms as that of: (B) Chikungunya is caused by a Gram
a) Taenia, a tapeworm negative bacterium
b) Wuchereria, a filarial worm (C) Tissue culture has proved useful in
c) Rhizopus, a mould obtaining virus free plants
d) Ascaris, a round worm (D) Beer is manufactured by distillation of
Ans. (c) fermented grape juice.
23. Which one of the following options gives the How many of the above statements are wrong?
correct matching of a disease with its a) Four b) One c) Two d) Three
causative organism and mode of infection? Ans. (d)
Disease Causative Mode of • Colostrum is rich in antibodies.
organisms Infection • Chikungunya is caused by Alphavirus
a) Typhoid Salmonella typhi With inspired air and transmitted to humans by Aedes
b) Pneumonia Streptococcus Droplet infection mosquito.
pneumoniae
• Beer and wine are produced without
c) Elephantiasi Wuchereria With infected water
s bancrofti and food
distillation; whereas whisky, brandy
Malaria Plasmodium Bite of male and rum are produced by distillation.
d)
vivax Anopheles Thus three statements are wrong.
mosquito
37
DR. ARVIND’S BIOLOGY CLASSES
(A Unit of Med-Xel Tutorials)
27. Identify the molecules (A) and (B) shows 28. Which one of the following statements is
below and select the right option giving their correct with respect to immunity?
source and use a) Antibodies are protein molecules, each
of which has four light chains
b) Rejection of a kidney graft is, the function
of B-lymphocytes
c) Preformed antibodies need to be injected
to treat the bite by a viper snake
d) The antibodies against small pox pathogen
are produced by T-lymphocytes
Ans. (c)

NEET – 2013

29. Infection of Ascaris usually occurs by:


a) Tse-tse fly.
b) mosquito bite.
Options:
c) drinking water containing eggs of
Molecule Source Use Ascaris.
a) (B) Cannabiniod Atropa Produces d) eating imperfectly cooked pork.
belladonna hallucinations Ans. (c)
b) (A) Morphine Papaver Sedative and 30. The cell-mediated immunity inside the
somniferum pain killer human body is carried out by:
c) (A) Cocaine Erythroxylum Accelerates
a) Thrombocytes b) Erythrocytes
coca the transport
c) T-lymphocytes d) B-lymphocytes
of dopamine
Ans. (c)
d) (B) Heroin Cannabis Depressant
sativa and slows
down body AIPMT – 2014
functions
Ans. (b) 31. Which is the particular type of drug that is
Hint: obtained from the plant whose one flowering
(a) Is the chemical structure of Morphine branch is shown below?
which is an effective sedative and
painkiller. It is extracted from Poppy plant
(Papaver somniferum)
(b) Is the chemical structure of Cannabinoid
molecule which is obtained from
Cannabis sativa and known for their
effects on the Cardiovascular system.
• Atropa belladonna and Datura plants
have hallucinogenic properties. a) Pain-killer b) Hallucinogen
• Cocaine is obtained from Erythroxylum c) Depressant d) Stimulant
coca and it interferes with the transport Ans. (b)
of dopamine in the CNS. Hint: The shown branch is of plant –
• Heroin is a derivative of morphine and is a “Datura” which gives us hallucinogens.
depressant and slows down body functions.
38
DR. ARVIND’S BIOLOGY CLASSES
(A Unit of Med-Xel Tutorials)
32. At which stage of HIV infection does one Hint: Ebola virus, spreads by direct contact
usually show symptoms of AIDS? with body fluids of an infected human or
a) When the viral DNA is produced by other animals. Semen or breast milk of a
transcriptase person after recovery from Ebola viral
b) Within 15 days of sexual contact with an disease may still carry the virus for several
infected person weeks to months. Fruit bats are believed
c) When the infected retro virus enters host to be the natural carrier of this virus.
cells Chikungunya virus is transmitted by
d) When HIV damages large number of Aedes, Culex and Mansonia mosquitoes.
helper T-lymphocytes. 38. Match each disease with its correct type of
Ans. (d) vaccine:
A) tuberculosis (i) harmless virus
AIPMT – 2015 B) whooping cough (ii) inactivated toxin
C) diphtheria (iii) killed bacteria
33. Which of the following endoparasites of D) polio (iv) harmless bacteria
humans does show viviparity? A B C D
a) Enterobius vermicularis a) (iii) (ii) (iv) (i)
b) Trichinella spiralis b) (iv) (iii) (ii) (i)
c) Ascaris Lumbricoides c) (i) (ii) (iv) (iii)
d) Ancylostoma duodenale d) (ii) (i) (iii) (iv)
Ans. (b) Ans. (b)
34. Which of the following is not a sexually
transmitted disease? AIPMT RETEST – 2015
a) Acquired Immuno Deficiency Syndrome
39. If you suspect major deficiency of
(AIDS)
antibodies in a person, to which of the
b) Trichomoniasis
following would you look for confirmatory
c) Encephalitis
evidence?
d) Syphilis
a) Fibrinogin in plasma
Ans. (c)
35. HIV that causes AIDS, first starts destroying: b) Serum albumins
a) Leucocytes c) Haemocytes
b) Helper T-Lymphocytes d) Serum globulins
c) Thrombocytes Ans. (d)
d) B-Lymphocytes 40. Which of the following immunoglobulins
Ans. (b) does constitute the largest percentage in
36. The active form of Entamoeba histolytica
human milk?
feeds upon:
a) Ig D b) Ig M c) Ig A d) Ig G
a) mucosa and submucosa of colon only
b) food in intestine Ans. (c)
c) blood only 41. Grafted kidney may be rejected in a patient
d) erythrocytes; mucosa and submucosa of due to:
colon a) Humoral immune response
Ans. (d) b) Cell-mediated immune response
c) Passive immune response
37. Which of the following viruses is not
transferred through semen of an infected d) Innate immune response
Ans. (b)
male?
42. Which of the following disease is caused
a) Human immunodeficiency virus
by a protozoan?
b) Chikungunya virus
a) Syphilis b) Influenza
c) Ebola virus
c) Babesiosis d) Blastomycosis
d) Hepatitis B virus
Ans. (c)
Ans. (b)
39
DR. ARVIND’S BIOLOGY CLASSES
(A Unit of Med-Xel Tutorials)
Hint:
• Syphilis is caused by bacteria 47. Which of the following is correct regarding
Treponema pallidum. AIDS causative agent HIV?
• Influenza is a viral disease. a) HIV is enveloped virus containing one
• Babesiosis – It is a tick transmitted zoonotic molecule of single-stranded RNA and
disease caused by protozoa of genus one molecule of reverse transcriptase.
babesia. The disease manifests like malaria b) HIV is enveloped virus that contains two
with fever, hemolytic anemia & identical molecules of single stranded
haemoglobinuria. RNA and two molecules of reverse
Blastomycosis – A fungal disease caused by transcriptase.
the fungus, Blastomyces dermatitidis a member c) HIV is unenveloped retrovirus.
of phylum Ascomycota. It can present in one of d) HIV does not escape but attacks the
the following ways – flu like illness, arthralgia, acquired immune response.
pneumonia, ARDS, skin lesions. Ans. (b)

AIPMT – 2016 NEET – 2017

43. In higher vertebrates, the immune system 53. MALT constitutes about______ percent of
can distinguish self-cells and non-self. If this the lymphoid tissue in human body.
property is lost due to genetic abnormality a) 50% b) 20% c) 70% d)
and it attacks self-cells, then it leads to: 10%
a) Graft rejection Ans. (a)
b) Auto-immune disease 54. Transplantation of tissues/organs fails often
c) Active immunity due to non-acceptance by the patient’s body.
d) Allergic response Which type of immune-response is
Ans. (b) responsible for such rejections?
44. Antivenom injection contains preformed a) Autoimmune response
antibodies while polio drops that are b) Cell-mediated immune response
administered into the body contain: c) Hormonal immune response
a) Harvested antibodies d) Physiological immune response
b) Gamma globulin Ans. (b)
c) Attenuated pathogens
d) Activated pathogens
Ans. (c)
45. Asthma may be attributed to:
a) Allergic reaction of the mast cells in the lungs
b) Inflammation of the trachea
c) Accumulation of fluid in the lungs
d) Bacterial infection of the lungs
Ans. (a)

NEET-2; 2016

46. Which of the following sets of diseases is


caused by bacteria?
a) Cholera and tetanus
b) Typhoid and smallpox
c) Tetanus and mumps
d) Herpes and influenza
Ans. (a)
40
DR. ARVIND’S BIOLOGY CLASSES
(A Unit of Med-Xel Tutorials)
STRATEGIES FOR ENHANCEMENT IN FOOD PRODUCTION a) Chilli b) Maize
c) Sugarcane d) Wheat
CBSE PRELIMS – 2009 Ans. (d)
1. Which one of the following has maximum 8. A collection of plants and seeds having diverse
genetic diversity in India? alleles of all the genes of a crop is called:
a) Wheat b) Tea a) Herbarium b) Germplasm
c) Teak d) Mango c) Gene library d) Genome
Ans. (d) Ans. (b)
Hint: Wheat originated in South-Western Asia. 9. Mutations can be induced with
Teak originated in Indonesia. The main a) Infra Red radiations
origin of Mango was in India. The Origin b) I A A
of Tea is said to be mainly in China and c) Ethylene
Burma. A plant shows maximum genetic d) Gamma radiations
diversity in its place of origin. Ans. (d)
2. Which of the following is not used as a 10. Which one of the following shows maximum
biopesticide? genetic diversity in India?
a) Nuclear Polyhedrosis Virus (NPV) a) Groundnut b) Rice
b) Xanthomonas campestris c) Maize d) Mango
c) Bacillus thuringiensis Ans. (b)
d) Trichoderma harzianum
Ans. (b) CBSE PRELIMS – 2012
3. Somaclones are obtained by:
a) Irradiation b) Genetic engineering 11. Which one of the following is a case of
c) Tissue culture d) Plant breeding wrong matching?
Ans. (c) a) Vector DNA – Site for t-RNA synthesis
4. Polyethylene glycol method is used for: b) Micropropagation – In vitro production of
a) Seedless fruit production plants in large numbers
b) Energy production from sewage c) Callus – Unorganised mass of cells
c) Gene transfer without a vector produced in tissue culture
d) Biodiesel production d) Somatic hybridization – Fusion of two
Ans. (c) diverse cells
Ans. (a)
12. Which part would be most suitable for raising
CBSE PRELIMS – 2010
virus-free plants for micropropagation?
a) Vascular tissue b) Meristem
5. Breeding of crops with high levels of
c) Node d) Bark
minerals, vitamins and proteins is called:
Ans. (b)
a) Micropropagation
b) Somatic hybridization
c) Biofortification CBSE MAINS – 2012
d) Biomagnification
Ans. (c) 13. Consider the following four statements (a–d)
and select the option which includes all the
correct ones only
CBSE PRELIMS – 2011
a) Single cell Spirulina can produce large
6. ”Jaya” and “Ratna” developed for green quantities of food rich in protein,
revolution in India are the varieties of: minerals, vitamins etc.
a) Maize b) Rice b) Body weight-wise the microorganism
c) Wheat d) Bajra Methylophilus methylotrophus may be
Ans. (b) able to produce several times more
7. ‘Himgiri’ developed by hybridisation and proteins than the cows per day
selection for disease resistance against rust c) Common button mushrooms are a very
pathogens is a variety of: rich source of vitamin C.
41
DR. ARVIND’S BIOLOGY CLASSES
(A Unit of Med-Xel Tutorials)
d) A rice variety has been developed which d) Both apical and axillary meristems
is very rich in calcium. Ans. (d)
Options
a) Statements (b), (c) and (d) AIPMT – 2015
b) Statements (a), (b)
c) Statements (c), (d) 18. Which of the following enhances or induces
d) Statements (a), (c) and (d) fusion of protoplasts?
Ans. (b) a) Polyethylene glycol and sodium nitrate
14. Green revolution in India occurred during: b) IAA and kinetin
a) 1980’s b) 1950’s c) 1960’s d) 1970’s c) IAA and gibberellins
Ans. (c) d) Sodium chloride and potassium chloride
Hint: The mid 1960’s saw the development of Ans. (a)
many high yielding varieties of wheat & 19. A technique of micropropagation is:
rice in India and this period is called the a) Somatic embryogenesis
Green Revolution. b) Protoplast fusion
c) Embryo rescue
NEET – 2013 d) Somatic hybridization
Ans.(a)
15. In plant breeding programmes, the entire Hint: Somatic embryogenesis is one step in
collection (of plants/seeds) having all the Micropropagation for regeneration of
diverse alleles for all genes in a given crop is plantlets. Here any cell of embryo sac or
called; any other part of sporophyte may give
a) evaluation and selection of parents rise to embryo. Embryo rescue, somatic
b) germplasm collection. hybridization (protoplast fusion) are
c) Selection of superior recombinants. advanced techniques which further may
d) Cross-hybridisation among the selected require the obtained plant to be induced
parents. to regenerate into whole plants.
Ans. (b)

AIPMT – 2014 AIPMT RETEST – 2015


20. Outbreeding is an important strategy of
16. In vitro clonal propagation in plants is
animal husbandry because it
characterized by:
a) Helps in accumulation of superior genes.
a) Microscopy
b) Is useful in producing purelines of
b) PCR and RAPD
animals
c) Northern blotting
c) Is useful in overcoming inbreeding
d) Electrophoresis and HPLC
depression
Ans. (b)
d) Exposes harmful recessive genes that
Hint:
are eliminated by selection
RAPD - Random amplification of
Ans. (c)
polymorphic DNA; it is similar to
PCR
HPLC - High performance liquid NEET-2; 2016
chromatography/ High pressure
liquid chromatography. 21. Interspecific hybridization is the mating of
17. To obtain virus – free healthy plants from a a) Animals within same breed without
diseased one by tissue culture technique, having common ancestors
which part/parts of the diseased plant will be b) Two different related species
taken? c) Superior males and females of different
a) Epidermis only breeds
b) Apical meristem only d) More closely related individuals within
c) Palisade parenchyma same breed for 4 – 6 generations
Ans. (b)
42
DR. ARVIND’S BIOLOGY CLASSES
(A Unit of Med-Xel Tutorials)
22. Among the following edible fishes, which one
is a marine fish having rich source of omega-
3 fatty acids?
a) Mystus b) Mangur
c) Mrigala d) Mackerel
Ans. (d)

NEET – 2017

23. Homozygous purelines in cattle can be


obtained by:
a) Mating of related individuals of same
breed.
b) Mating of unrelated individuals of same
breed.
c) Mating of individuals of different breed.
d) Mating of individuals of different species.
Ans. (a)
43
DR. ARVIND’S BIOLOGY CLASSES
(A Unit of Med-Xel Tutorials)

MICROBES IN HUMAN WELFARE CBSE PRELIMS – 2011

CBSE PRELIMS – 2009 7. A prokaryotic autotrophic nitrogen fixing


symbiont is found in:
1. Which one of the following pairs is wrongly a) Alnus b) Cycas
matched? c) Cicer d) Pisum
a) Fruit juice – pectinase Ans. (b)
b) Textile – amylase Hint: Cicer and Pisum belong to family
c) Detergents – lipase Leguminosae, where there is association
d) Alcohol – nitrogenase of free living bacteria Rhizobium.
Ans. (d) In Alnus there is association of free
2. Which of the following is a symbiotic nitrogen living bacteria Frankia.
fixer? In Cycas there is association of
a) Frankia b) Azolla Cyanobacteria which are prokaryotic
c) Glomus d) Azotobacter autotrophic nitrogen fixing organisms.
Ans. (a) 8. The most common substrate used in
Hint: Frankia is a nitrogen fixing bacterium distilleries for the production of ethanol is:
associated with the root nodules of a) Corn meal b) Soya meal
nonlegume plants. Azolla is a fern which c) Ground gram d) Molasses
shows symbiotic association with blue Ans. (d)
green alga Anabaena. Azotobacter is a 9. Which one of the following is not a biofertilizer?
free living nitrogen fixing bacterium. a) Agrobacterium b) Rhizobium
Glomus is the most common fungal c) Nostoc d) Mycorrhiza
partner of mycorrhiza. Ans. (a)
10. Secondary sewage treatment is mainly a:
CBSE PRELIMS – 2010 a) Physical process
b) Mechanical process
3. Select the correct statement from the following: c) Chemical process
a) Activated sludge-sediment in settlement d) Biological process
tanks of sewage treatment plant is a rich Ans. (d)
source of aerobic bacteria 11. Which of the following is mainly produced by
b) Biogas is produced by the activity of the activity of anaerobic bacteria on sewage?
aerobic bacteria on animal waste a) Laughing gas b) Propane
c) Methanobacterium is an aerobic c) Mustard gas d) Marsh gas
bacterium found in rumen of cattle Ans. (d)
d) Biogas, commonly called gobar gas, is 12. Ethanol is commercially produced through a
pure methane particular species of
Ans. (a) a) Saccharomyces b) Clostridium
4. Which one of the following is not used in c) Trichoderma d) Aspergillus
organic farming? Ans. (a)
a) Snail b) Glomus 13. Which one of the following helps in absorption
c) Earthworm d) Oscillatoria of phosphorus from soil by plants?
Ans. (a) a) Glomus b) Rhizobium
5. A common biocontrol agent for the control of c) Frankia d) Anabaena
plant diseases is: Ans. (a)
a) Trichoderma b) Baculovirus
c) Bacillus thuringiensis d) Glomus
Ans. (a)
6. The common nitrogen-fixer in paddy fields is:
a) Frankia b) Rhizobium
c) Azospirillum d) Oscillatoria
Ans. (d)
44
DR. ARVIND’S BIOLOGY CLASSES
(A Unit of Med-Xel Tutorials)
CBSE PRELIMS – 2012 18. A patient brought to a hospital with myocardial
infarction is normally immediately given:
14. Yeast is used in the production of
a) Streptokinase b) Cyclosporin-A
a) Lipase and pectinase
b) Bread and beer c) Statins d) Penicillin
c) Cheese and butter Ans. (a)
d) Citric acid and lactic acid 19. The most abundant prokaryotes helpful to
Ans. (b) humans in making curd from milk and in
15. Which one of the following is an example of production of antibiotics are the ones
carrying out biological control of categorised as:
pests/diseases using microbes? a) Archaebacteria
a) Nucleopolyhedrovirus against white rust
in Brassica b) Chemosynthetic autotrophs
b) Bt-cotton to increase cotton yield c) Heterotrophic bacteria
c) Lady bird beetle against aphids in mustard d) Cyanobacteria
d) Trichoderma sp. against certain plant Ans. (c)
pathogens
Ans. (d) CBSE MAINS – 2010
Hint: Trichoderma sp. are free-living fungi.
Nucleopolyhedroviruses mainly attack 20. An example of endomycorrhiza is
insects and pathogens. Bt-cotton also a) Nostoc b) Glomus
seems to be a correct choice, but since c) Agaricus d) Rhizobium
the pest is not mentioned in the choice, Ans. (b)
this is probably not the best choice.
Ladybirds are used to control aphids, but CBSE MAINS – 2011
they are not microbes.
16. Monascus purpureus is a yeast used 21. Read the following statement having two
commercially in the production of: blanks (A and B):
a) streptokinase for removing clots from the “A drug used for ___(A)__ patients is obtained
blood vessels from a species of the organism ___(B)___.”
b) citric acid The one correct option for the two blanks is:
Blank – A Blank – B
c) blood cholesterol lowering statins
a) Heart Penicillium
d) ethanol b) Organ-transplant Trichoderma
Ans. (c) c) Swine flu Monascus
17. Which one of the following microbes forms d) AIDS Pseudomonas
symbiotic association with plants and helps Ans. (b)
them in their nutrition? Hint: Cyclosporin A (immunosupressive agent)
a) Aspergillus b) Glomus is produced by fungus Trichoderma
polysporum.
c) Trichoderma d) Azotobacter 22. Consider the following statements (A-D)
Ans. (b) about organic farming:
Hint: Trichoderma are free-living fungi and are (A) Utilizes genetically modified crops like Bt cotton
effective biocontrol agents of several plant (B) Uses only naturally produced inputs like
pathogens. Azotobacter is a free living compost
nitrogen fixing bacteria. Many members of (C) Does not use pesticides and urea
genus Glomus form mycorrhiza. (D) Produces vegetables rich in vitamins and
minerals
Aspergillus is the common green mould
Which of the above statements are correct?
belonging to the ascomycetes. a) (B), (C) and (D) b) (C) and (D) only
c) (B) and (C) only d) (A) and (B) only
Ans. (c)
45
DR. ARVIND’S BIOLOGY CLASSES
(A Unit of Med-Xel Tutorials)
23. Which one of the following is a wrong matching AIPMT RETEST – 2015
of a microbe and its industrial product, while the
remaining three are correct?
28. Match the following list of microbes and their
a) Yeast – statins
b) Acetobacter aceti – acetic acid importance:
c) Clostridium butylicum – lactic acid (A) Sacharomyc (i) Production of
d) Aspergillus niger – citric acid es immunospuppressiv
Ans. (c) cerevisiae e agents
Hint: Clostridium butylicum yields butyric acid. (B) Monascus (ii)
Ripening of Swiss
Lactic acid is obtained from Lactobacillus. purpureus cheese
Statins are obtained from the yeast – (C) Trichoderma (iii) Commercial
Monascus purpureus. polysporum production of
ethanol
CBSE MAINS – 2012 (D) Propionibact (iv) Production of
erium blood-cholesterol
24. In gobar gas, the maximum amount is that of: sharmanii lowering agents
a) Propane b) Carbon dioxide (A) (B) (C) (D)
c) Butane d) Methane a) (iii) (iv) (i) (ii)
Ans. (d) b) (iv) (iii) (ii) (i)
c) (iv) (ii) (i) (iii)
NEET – 2013 d) (iii) (i) (iv) (ii)
Ans. (a)
25. A good producer of citric acid is:
a) Clostridium b) Saccharomyces
c) Aspergillus d) Pseudomonas AIPMT – 2016
Ans. (c)
26. During sewage treatment, biogases are
produced which include: 29. Which of the following is wrongly matched in
a) hydrogensulphide, methane, sulphur the given table?
dioxide Microbe Product Application
b) hydrogensulphide, nitrogen, a) Monascus Statins Lowering of
methane purpureus blood
c) methane, hydrogensulphide, Carbon cholesterol
dioxide b) Streptococcus Streptokinase Removal of
clot from
d) methane, oxygen, hydrogensulphide
blood vessel
Ans. (c) c) Clostridium Lipase Removal of
butylicum oil stains
AIPMT – 2014 d) Trichoderma Cyclosporin A Immuno
polysporum suppressive
27. What gases are produced in anaerobic drug
sludge digesters? Ans. (c)
a) Hydrogen Sulphide and CO2
b) Methane and CO2 only
c) Methane, Hydrogen Sulphide and CO 2
d) Methane, Hydrogen Sulphide and O 2
Ans. (c)
46
DR. ARVIND’S BIOLOGY CLASSES
(A Unit of Med-Xel Tutorials)
NEET-2; 2016
30. Match Column – I with Column – II and
select the correct option using the codes
given below:
Column – I Column – II
1. Citric acid (i) Trichoderma
2. Cyclosporin A (ii) Clostridium
3. Statins (iii) Aspergillus
4. Butyric acid (iv) Monascus
Codes
1 2 3 4
a) (iii) (i) (ii) (iv)
b) (iii) (i) (iv) (ii)
c) (i) (iv) (ii) (iii)
d) (iii) (iv) (i) (ii)
Ans. (b)

NEET – 2017

32. Which of the following in sewage treatment


removes suspended solids?
a) Tertiary treatment
b) Secondary treatment
c) Primary treatment
d) Sludge treatment
Ans. (c)
33. Which of the following is correctly matched
for the product produced by them?
a) Acetobacter aceti : Antibiotics
b) Methanobacterium : Lactic acid
c) Penicillium notatum : Acetic acid
d) Sacchromyces cerevisiae : Ethanol
Ans. (d)
47
DR. ARVIND’S BIOLOGY CLASSES
(A Unit of Med-Xel Tutorials)

BIOTECHNOLOGY CBSE PRELIMS – 2010


6. Stirred-tank bioreactors have been designed for:
CBSE PRELIMS – 2009 a) Availability of oxygen throughout the process
1. The bacterium Bacillus thuringiensis is b) Addition of preservatives to the product
widely used in contemporary biology as: c) Purification of the product
a) Agent for production of dairy products d) Ensuring anaerobic conditions in the
b) Source of industrial enzyme culture vessel
c) Indicator of water-pollution Ans. (a)
d) Insecticide 7. Which one of the following is used as vector
Ans. (d) for cloning genes into higher organisms?
2. Which one of the following is commonly used a) Retrovirus
in transfer of foreign DNA into crop plants? b) Baculovirus
a) Agrobacterium tumefaciens c) Salmonella typhimurium
b) Penicillium expansum d) Rhizopus nigricans
c) Trichoderma harzianum Ans. (a)
d) Meloillogyne incognita 8. The genetically-modified (GM) brinjal in India
Ans. (a) has been developed for:
3. Transgenic plants are the ones: a) Drought-resistance
a) Produced after protoplast fusion in b) Insect-resistance
artificial medium. c) Enhancing shelf life
b) Grown in artificial medium after d) Enhancing mineral content
hybridization in the field Ans. (b)
c) Produced by a somatic embryo in 9. Genetic engineering has been successfully
artificial medium used for producing:
d) Generated by introducing foreign DNA a) Animals like bulls for farm work as they
into a cell and regenerating a plant from have super power
that cell. b) Transgenic mice for testing safety of
Ans. (d) polio vaccine before use in humans
4. The genetic defect-adenosine deaminase c) Transgenic models for studying new
(ADA) deficiency may be cured permanently by treatments for certain cardiac diseases
a) Introducing bone marrow cells producing d) Transgenic Cow-Rosie which produces
ADA into cells at early embryonic stages. high fat milk for making ghee
b) Enzyme replacement therapy Ans. (b)
c) Periodic infusion of genetically engineered 10. Restriction endonucleases are enzymes which:
lymphocytes having functional ADA cDNA. a) Remove nucleotides from the ends of the
d) Administering adenosine deaminase DNA molecule
activator. b) Make cuts at specific positions within the
Ans. (a) DNA molecule
5. What is true about Bt toxin? c) Recognize a specific nucleotide
a) The activated toxin enters the ovaries of sequence for binding of DNA ligase
the pest to sterilize it and thus prevent d) Restrict the action of the enzyme DNA
its multiplication. polymerase
b) The concerned Bacillus has antitoxins Ans. (b)
c) The inactive protoxins gets converted 11. Some of the characteristics of Bt cotton are:
into active form in the insect gut a) High yield and resistance to bollworms
d) Bt protein exists as active toxin in the b) Long fibre and resistance to aphids
Bacillus. c) Medium yield, long fibre and resistance
Ans. (c) to beetle pests
d) High yield and production of toxic protein
crystals which kill dipteran pests
Ans. (a)
48
DR. ARVIND’S BIOLOGY CLASSES
(A Unit of Med-Xel Tutorials)
12. Which one of the following palindromic base 18. The process of RNA interference has been
sequences in DNA can be easily cut at about the used in the development of plants resistant to:
middle by some particular restriction enzyme? a) Nematodes b) Fungi
a) 5’ _______ CACGTA _______ 3’ c) Viruses d) Insects
3’ _______ CTCAGT _______ 5’ Ans. (a)
b) 5’ _______ CGTTCG _______ 3’
3’ _______ ATGGTA _______ 5’
c) 5’ _______ GATATG _______ 3’ CBSE PRELIMS – 2012
3’ _______ CTACTA _______ 5’
19. Which one is a true statement regarding
d) 5’ _______ GAATTC _______ 3’
DNA polymerase used in PCR?
3’ _______ CTTAAG _______ 5’
a) It serves as a selectable marker
Ans. (d)
b) It is isolated from a virus
13. An improved variety of transgenic basmati rice:
c) It remains active at high temperature
a) Gives high yield but has no characteristic
d) It is used to ligate introduced DNA in
aroma
recipient cells
b) Does not require chemical fertilizers and
Ans. (c)
growth hormones
20. For transformation, micro-particles coated
c) Gives high yield and is rich in vitamin A
with DNA to be bombarded with gene gun
d) Is completely resistant to all insect pests
are made up of:
and diseases of paddy
a) Platinum or Zinc b) Silicon or Platinum
Ans. (c)
c) Gold or Tungsten d) Silver or Platinum
Ans. (c)
CBSE PRELIMS – 2011 21. The figure below is the diagrammatic
representation of the E. Coli vector pBR 322.
14. Given below is a sample of a portion of DNA Which one of the given options correctly
strand giving the base sequence on the opposite identifies its certain component(s)?
strands. What is so special shown in it?
5 _______GAATTC________3
3________CTTAAG_______5
a) Replication completed
b) Deletion mutation
c) Start codon at the 5 end
d) Palindromic sequence of base pairs
Ans. (d)
15. There is a restriction endonuclease called
EcoRI. What does “co” part in it stand for?
a) Colon b) Coelom a) rop-reduced osmotic pressure
c) Coenzyme d) Coli b) Hind III, EcoRI-selectable markers
Ans. (d) c) ampR, tetR-antibiotic resistance genes
16. Maximum number of existing transgenic d) ori-original restriction enzyme
animals is of: Ans. (c)
a) Fish b) Mice 22. Consumption of which one of the following
c) Cow d) Pig foods can prevent the kind of blindness
Ans. (b) associated with vitamin ‘A’ deficiency?
17. Agarose extracted from sea weeds finds use in: a) Canolla b) Golden rice
a) Spectrophotometry c) Bt-Brinjal d) ‘Flaver Savr’ tomato
b) Tissue Culture Ans. (b)
c) PCR
d) Gel electrophoresis
Ans. (d)
49
DR. ARVIND’S BIOLOGY CLASSES
(A Unit of Med-Xel Tutorials)
CBSE MAINS – 2010 28. In history of biology, human genome project
led to the development of:
23. Which one of the following is now being a) Biotechnology b) Biomonitoring
commercially produced by biotechnological c) Bioinformatics d) Biosystematics
procedures? Ans. (c)
a) Nicotine b) Morphine 29. Which one of the following techniques made it
c) Quinine d) Insulin possible to genetically engineer living
Ans. (d) organisms?
24. In genetic engineering, a DNA segment a) Recombinant DNA techniques
(gene) of interest, is transferred to the host b) X-ray diffraction
cell through a vector. Consider the following c) Heavier isotope labelling
four agents (A – D) in this regard and select d) Hybridization
the correct option about which one or more Ans. (a)
of these can be used as a vector/vectors: 30. Read the following four statements (A-D)
(A) A bacterium (B) Plasmid about certain mistakes in two of them.
(C) Plasmodium (D) Bacteriophage (A) The first transgenic buffalo, Rosie
Options: produced milk which was human alpha-
a) (A), (B) and (D) only lactalbumin enriched
b) (A) only (B) Restriction enzymes are used in isolation
c) (A) and (C) only of DNA from other macromolecules
d) (B) and (D) only (C) Downstream processing is one of the
Ans. (d) steps of R-DNA technology
25. Which of the following are used in gene (D) Disarmed pathogen vectors are also used
cloning? in transfer of R-DNA into the host
a) Nucleoids b) Lomasomes Which are the two statements having mistakes?
c) Mesosomes d) Plasmids a) Statements (B) and (C)
Ans. (d) b) Statements (C) and (D)
c) Statements (A) and (C)
CBSE MAINS – 2011 d) Statements (A) and (B)
Ans. (d)
26. Bacillus thuringiensis forms protein crystals Hint: (A) Rosie was a transgenic cow.
which contain insecticidal protein (B) Restriction enzymes help in cutting
This protein: the DNA itself.
a) Binds with epithelial cells of midgut of the
insect pest ultimately killing it.
b) Is coded by several genes including the CBSE MAINS – 2012
gene cry
31. Tobacco plants resistant to a nematode have
c) Is activated by acid pH of the foregut of
been developed by the introduction of DNA
the insect pest
that produced (in the host cells):-
d) Does not kill the carrier bacterium which is
a) An antifeedant
itself resistant to this toxin
b) A toxic protein
Ans. (a)
c) Both sense and anti-sense RNA
Hint: Bt toxin is activated by alkaline pH of the
gut. In bacteria it exists as inactive
d) A particular hormone
protoxin and that is why it does not harm Ans. (c)
the bacteria. 32. In genetic engineering, the antibiotics are used:
27. Silencing of mRNA has been used in a) As sequences from where replication starts
producing transgenic plants resistant to: b) To keep the cultures free of infection
a) Bollworms b) Nematodes c) As selectable markers
c) White rusts d) Bacterial blights d) To select healthy vectors
Ans. (b) Ans. (c)
50
DR. ARVIND’S BIOLOGY CLASSES
(A Unit of Med-Xel Tutorials)
33. The first clinical gene therapy was given for 37. The colonies of recombinant bacteria
treating: appear white in contrast to blue colonies of
a) Rheumatoid arthritis non-recombinant bacteria because of:
b) Adenosine deaminase deficiency a) Insertional inactivation of
c) Diabetes mellitus alphagalactosidase in recombinant bacteria
d) Chicken pox b) Inactivation of glycosidase enzyme in
Ans. (b) recombinant bacteria
34. The figure below shows three steps (A, B, C) c) Non-recombinant bacteria containing beta-
of Polymerase Chain Reaction (PCR). galactosidase
Select the option giving correct identification d) Insertional inactivation of
together with what it represents? alphagalactosidase in non-recombinant
bacteria
Ans. CBSE has given the answer (a) but the
correct answer is (c) as per NCERT.
Hint: In insertional inactivation a recombinant
DNA is inserted within the coding sequence
of enzyme  – galactosidase. If the plasmid
in the bacteria does not have the insert (i.e.,
non – recombinants) then the chromogenic
substrate gives blue coloured colonies.
Presence of the insert (i.e., recombinants)
leads to inactivation of the -galactosidase
enzyme thus the colonies do not produce
Options:
any colour.
a) C-Extension in the presence of heat
Option:
stable DNA polymerase
b) A-Annealing with two set of primers • Option (a) is wrong, it should be 
c) B-Denaturation at a temperature of about galactosidase
98°C separating the two DNA strands • Option (b) is wrong, it should be
d) A-Denaturation at a temperature of about galactosidase instead of glycosidase
50°C • Option (c) is correct, as the non-
Ans. (a) recombinants have the  galactosidase
35. Biolistics (gene-gun) is suitable for: enzymes intact.
a) Constructing recombinant DNA by joining • Option (d) is wrong it should be
with vectors “Insertional inactivation of 
b) DNA finger printing galactosidase of recombinant bacteria”.
c) Disarming pathogen vectors 38. DNA fragments generated by the restriction
d) Transformation of plants cells endonucleases in a chemical reaction can
Ans. (d) be separated by:
a) Electrophoresis
b) Restriction mapping
NEET – 2013 c) Centrifugation
d) Polymerase chain reaction
36. Which of the following Bt crops is being Ans. (a)
grown in India by the farmers? 39. Which of the following is not correctly
a) Brinjal b) Soybean matched for the organism and its cell wall
c) Maize d) Cotton degrading enzyme?
Ans. (d) a) Algae – Methylase
b) Fungi – Chitinase
c) Bacteria – Lysozyme
d) Plant cells – Cellulase
Ans. (a)
51
DR. ARVIND’S BIOLOGY CLASSES
(A Unit of Med-Xel Tutorials)
AIPMT – 2014 44. Which body of the Government of India
regulates GM research and safety of
40. Which vector can clone only a small introducing GM organisms for public
fragment of DNA? services?
a) Cosmid a) Indian Council of Agricultural Research
b) Bacterial artificial chromosome b) Genetic Engineering Approval Committee
c) Yeast artificial chromosome c) Research Committee on Genetic Manipulation
d) Plasmid d) Bio-safety committee
Ans. (d) Ans. (b)
Hint:
• Cosmid - Can clone DNA fragments upto AIPMT RETEST – 2015
45 kb in length
• BAC - can accommodate upto 300 – 45. The cutting of DNA at specific locations
350kb of foreign DNA became possible with the discovery of:
• YAC - used for cloning large (upto a) Restriction enzymes b) Probes
1000kb or 1 Mb) DNA
c) Selectable markers d) Ligases
segments.
• Plasmids - They can accommodate < 5 – 8 Ans. (a)
kb DNA inserts. 46. The introduction of t-DNA into plants
41. The first human hormone produced by involves:
recombinant DNA technology is: a) Infection of the plant by Agrobacterium
a) Progesterone b) Insulin tumefaciens
c) Estrogen d) Thyroxin b) Altering the pH of the soil, then heat-
Ans. (b) shocking the plants
c) Exposing the plants to cold for a brief
period
AIPMT – 2015 d) Allowing the plant roots to stand in water
Ans. (a)
42. The crops engineered for glyphosate are 47. The DNA molecule to which the gene of
resistant / tolerant to: interest is integrated for cloning is called:
a) Bacteria b) Insects a) Transformer b) Vector
c) Herbicides d) Fungi c) Template d) Carrier
Ans. (c) Ans. (b)
Hint: Glyphosate is a broad spectrum
48. Golden rice is a genetically modified crop
herbicide used to kill weeds which are
known to compete with commercial plant where the incorporated gene is meant
crops. Thus crops ressistant to for biosynthesis of:
glyphosate allow farmers to use this as a a) Vitamin B b) Vitamin C
weedicide without damaging the cereal c) Omega 3 d) Vitamin A
crops/commercial crops. Ans. (d)
43. In Bt cotton, the Bt toxin present in plant
tissue as pro-toxin is converted into active AIPMT – 2016
toxin due to:
a) Acidic pH of the insect gut 49. The two polypeptides of human insulin are
b) Action of gut micro-organisms linked together by:
c) Presence of conversion factors in insect gut a) Phosphodiester bond
d) Alkaline pH of the insect gut b) Covalent bond
Ans. (d) c) Disulphide bridges
d) Hydrogen bonds
Ans. (c)
52
DR. ARVIND’S BIOLOGY CLASSES
(A Unit of Med-Xel Tutorials)
50. The taq polymerase enzyme is obtained from: NEET – 2017
a) Thiobacillus ferroxidans
b) Bacillus subtilis 58. The process of separation and purification of
c) Pseudomonas putida expressed protein before marketing is called
d) Thermus aquaticus a) Upstream processing
Ans. (d) b) Downstream processing
51. Which part of the tobacco plant is infected by
c) Bioprocessing
Meloidogyne incognita?
a) Leaf b) Stem d) Postproduction processing
c) Root d) Flower Ans. (b)
Ans. (c) 59. What is the criterion for DNA fragments
52. Which of the following is a restriction movement on agarose gel during gel
endonuclease? electrophoresis?
a) Protease b) DNase I a) The larger the fragment size, the farther
c) RNase d) Hind II it moves
Ans. (d) b) the smaller the fragment size, the farther
NEET-2; 2016 it moves
c) positively charged fragments move to
53. Stirred-tank bioreactors have been designed for
a) Purification of product farther end
b) Addition of preservatives to the product d) negatively charged fragments do not
c) Availability of oxygen throughout the process move
d) Ensuring anaerobic conditions in the Ans. (b)
culture vessel 60. A gene whose expression helps to identify
Ans. (c) transformed cell is known as:
54. A foreign DNA and plasmid cut by the same a) Selectable marker b) Vector
restriction endonuclease can be joined to c) Plasmid d) Structural gene
form a recombinant plasmid using Ans. (a)
a) Eco RI b) Taq polymerase 61. The DNA fragments separated on an
c) Polymerase III d) Ligase agarose gel can be visualized after staining
Ans. (d) with:
55. Which of the following is not a component of a) Bromophenol blue
downstream processing? b) Acetocarmine
a) Separation b) Purification c) Aniline blue
c) Preservation d) Expression d) Ethidium bromide
Ans. (d) Ans. (d)
56. Which of the following restriction enzymes
produces blunt ends?
a) Sal I b) Eco RV
c) Xho I d) Hind III
Ans. (b)
57. Which kind of therapy was given in 1990 to a
four-year-old girl with adenosine deaminase
(ADA) deficiency?
a) Gene therapy
b) Chemotherapy
c) Immunotherapy
d) Radiation therapy
Ans. (a)
53
DR. ARVIND’S BIOLOGY CLASSES
(A Unit of Med-Xel Tutorials)

ECOLOGY
CBSE PRELIMS – 2010
ORGANISMS & POPULATIONS
3. Study the four statements given below and
select the two correct ones out of them:
CBSE PRELIMS – 2009 (i) A lion eating a deer and a sparrow
1. A country with a high rate of population feeding on grain are ecologically similar
growth took measures to reduce it. The in being consumers
figure below shows age-sex pyramids of (ii) Predator star fish Pisaster helps in
populations A and B twenty years apart. maintaining species diversity of some
Select the correct interpretation about them: invertebrates
(iii) Predators ultimately lead to the extinction
of prey species
(iv) Production of chemicals such as nicotine,
strychnine by the plants are metabolic
disorders
The two correct statements are:
a) (i) and (ii) b) (ii) and (iii)
c) (iii) and (iv) d) (i) and (iv)
Ans. (a)
4. The figure given below is a diagrammatic
representation of response of organisms to
abiotic factors. What do a, b and c represent
respectively?

A B C
a) Regulator Conformer Partial regulator
b) Conformer Regulator Partial regulator
Interpretations: c) Regulator Partial Conformer
(a) “B” is more recent showing that regulator
population is very young. d) Partial Regulator conformer
(b) “A” is the earlier pyramid and no regulator
change has occurred in the growth rate. Ans. (a)
(c) “A” is more recent and shows slight
reduction in the growth rate.
(d) “B” is earlier pyramid and shows
stabilized growth rate.
Ans. (c)
2. Reduction in vascular tissue, mechanical
tissue and cuticle is characteristic of:
a) Epiphytes b) Hydrophytes
c) Xerophytes d) Mesophytes
Ans. (b)
54
DR. ARVIND’S BIOLOGY CLASSES
(A Unit of Med-Xel Tutorials)
CBSE PRELIMS – 2011 CBSE MAINS PMT – 2010
9. Which one of the following is most
5. What type of human population is appropriately defined?
represented by the following age pyramid? a) Host is an organism which provides food
to another organism
b) Amensalism is a relationship in which
one species is benefited whereas the
other is unaffected
c) Predator is an organism that catches and
kills other organism for food
d) Parasite is an organism which always
a) Vanishing population lives inside the body of other organism
b) Stable population and may kill it
c) Declining population Ans. (c)
d) Expanding population
Ans. (c) CBSE MAINS PMT– 2011
6. Of the total incident solar radiation the
10. Consider the following statements (A) – (D)
proportion of PAR is:
each with one or two blanks
a) About 70% b) About 60%
(A) Bears go into ___(1)____ during winter to
c) Less than 50% d) More than 80%
___(2)__ cold weather.
Ans. (c)
(B) A conical age pyramid with a broad base
7. Consider the following four conditions (a – d) and
represents ___(3)____ human population
select the correct pair of them as adaptation to
(C) A wasp pollinating a fig flower is an
environment in desert lizards. The conditions:
example of ___(4)____.
(i) Burrowing in soil to escape high
(D) An area with high levels of species
temperature
richness is known as ___(5)____.
(ii) Losing heat rapidly from the body during
Which one of the following options, gives the
high temperature
correct fill ups for the respective blank
(iii) Bask in sun when temperature is low
numbers from (1) to (5) in the statements?
(iv) Insulating body due to thick fatty dermis
a) (3) – stable (4) – commensalism,
Options:
a) (iii), (iv) b) (i), (iii) (5) marsh
b) (1) – aestivation, (2) – escape, (3) – stable,
c) (ii), (iv) d) (i), (ii)
(4) – mutualism
Ans. (b)
c) (3) – expanding, (4) – commensalism,
8. Which one of the following is categorised as
(5) – biodiversity park
a parasite in true sense?
d) (1) – hibernation, (2) – escape,
a) The female Anopheles bites and sucks
(3) – expanding, (5) hot spot.
blood from humans.
Ans. (d)
b) Human foetus developing inside the uterus
Hint:1. Hibernation 2. Escape
draws nourishment from the mother 3. Expanding 4. Mutualism
c) Head louse living on the human scalp as 5. Hot spot
well as laying eggs on human hair 11. The logistic population growth is expressed
d) The cuckoo (koel) lay its eggs in crow’s nest by the equation:
Ans. (c)
K N K N
Hint: A parasite in the true sense is an a) dt/dN = Nr   b) dN/dt = rN  
organism which obtains both food and  K   K 
space/shelter from the host and does not NK 
benefit the host. c) dN/dt = rN d) dN/dt = rN  
 N 
Ans. (b)
55
DR. ARVIND’S BIOLOGY CLASSES
(A Unit of Med-Xel Tutorials)
NEET – 2013
12. Natural reservoir of phosphorus is:
a) Rock b) Fossils
c) Sea water d) Animal bones
Ans. (a)
13. A sedentary sea anemone gets attached to the
shell lining of hermit crab. The association is:
a) Commensalism b) Amensalism a) Population B competed more
c) Ectoparasitism d) Symbiosis successfully for food than population A
Ans. (a) b) Population A produced more offspring
Hint: Some authors have called this than population B
association Mutualism (Symbiosis), some c) Population A consumed the members of
called it Protocooperation while some population B
others say it is Commensalism. Thus (a) d) Both plant populations in this habitat
and (d) are both correct. decreased
14. A biologist studied the population of rats in a Ans. (a)
barn. He found that the average natality was
250, average mortality 240, immigration 20 AIPMT – 2015
and emigration 30. The net increase in
population is: 18. The species confined to a particular region
a) 05 b) zero c) 10 d) 15 and not found elsewhere is termed as:
Ans. (b) a) Keystone b) Alien
c) Endemic d) Rare
AIPMT – 2014 Ans. (c)
19. An association of individuals of different
15. Just as a person moving from Delhi to species living in the same habitat and having
Shimla, escape the heat for the duration of functional interactions is:
hot summer, thousands of migratory birds a) Ecological niche b) Biotic community
from Siberia and other extremely cold c) Ecosystem d) Population
northern regions move to: Ans. (b)
a) Keolado National Park 20. In which of the following interactions both
b) Western Ghat partners are adversely affected?
c) Meghalaya a) Competition b) Predation
d) Corbett National Park c) Parasitism d) Mutualism
Ans. (a)
Ans. (a)

AIPMT – 2015 AIPMT – 2016

16. Most animals are tree dwellers in a: 21. Gause’s principle of competitive exclusion
a) thorn woodland states that:
b) temperate deciduous forest a) Competition for the same resources
c) tropical rain forest excludes species having different food
d) coniferous forest preferences
Ans. (c) b) No two species can occupy the same
niche indefinitely for the same limiting
17. The following graph depicts changes in two resources
populations (A and B) of herbivores in a
c) Larger organisms exclude smaller ones
grassy field. A possible reason for these through competition
changes is that: d) More abundant species will exclude the
less abundant species through competition
Ans. (b)
56
DR. ARVIND’S BIOLOGY CLASSES
(A Unit of Med-Xel Tutorials)
22. When does the growth rate of a population 27. Plants which produce characteristic
following the logistic model equal zero? The pneumatophores and show vivipary belong
logistic model is given as dN /dt = rN (1- to:
N/K): a) Mesophytes b) Halophytes
a) When N nears the carrying capacity of c) Psammophytes d) Hydrophytes
the habitat. Ans. (b)
b) When N / K equals zero.
c) When death rate is greater than birth
rate.
d) When N / K is exactly one.
Ans. (d)

NEET-2; 2016
23. Which of the following is correct for r-
selected species?
a) Large number of progeny with small size
b) Large number of progeny with large size
c) Small number of progeny with small size
d) Small number of progeny with large size
Ans. (a)
Hint: r-selected species are those that have a
high growth rate; large number of
offsprings, small body size, early maturity,
short generation time and ability to
disperse offspring widely.
24. If ‘+’ sign is assigned to beneficial
interaction, ‘-’ sign to detrimental and ‘O’ sign
to neutral interaction, then the population
interaction represented by ‘+’ ‘-’ refers to
a) Mutualism b) Amensalism
c) Commensalism d) Parasitism
Ans. (d)
25. The principle of competitive exclusion was
stated by
a) C. Darwin b) G. F. Gause
c) MacArthur d) Verhulst and Pearl
Ans. (b)

NEET- 2017
26. Asymptote in a logistic growth curve is
obtained when:
a) The value of ‘r’ approaches zero
b) K = N
c) K > N
d) K < N
Ans. (b)
57
DR. ARVIND’S BIOLOGY CLASSES
(A Unit of Med-Xel Tutorials)

ECOSYSTEM d) It is similar to primary succession except


that it has a relatively fast pace.
Ans. (b)
CBSE PRELIMS – 2009 7. Which one of the following statements for
1. Which one of the following types of pyramid of energy is incorrect, whereas the
organisms occupy more than one trophic remaining three are correct?
level in a pond ecosystem? a) Its base is broad
a) Zooplankton b) Frog b) It shows energy content of different
c) Phytoplankton d) Fish trophic level organisms
Ans. (b) c) It is inverted in shape
Hint: Tadpole is herbivorous. Adult frog is d) It is upright in shape
carnivorous. Ans. (c)
2. The correct sequence of plants in a
hydrosere is: CBSE PRELIMS – 2012
a) Pistia  Volvox  Scirpus  Hydrilla 
Oak  Lantana 8. Identify the possible link “A” in the following
b) Oak  Lantana  Volvox  Hydrilla  food chain:
Pistia  Scirpus Plant  insect  frog  “A”  Eagle
c) Oak  Lantana  Scirpus  Pistia  a) Wolf b) Cobra
Hydrilla  Volvox c) Parrot d) Rabbit
d) Volvox  Hydrilla  Pistia  Scirpus  Ans. (b)
Lantana  Oak 9. The upright pyramid of number is absent in:
Ans. (d) a) Forest b) Lake
c) Grassland d) Pond
Ans. (a)
CBSE PRELIMS – 2010 10. Given below is an imaginary pyramid of
numbers. What could be one of the
3. The biomass available for consumption by the possibilities about certain organisms at some
herbivores and the decomposers is called: of the different levels?
a) Gross primary productivity
b) Net primary productivity
c) Secondary productivity
d) Standing crop
Ans. (d)
4. Which one of the following is one of the
characteristics of a biological community?
a) Sex-ratio b) Stratification
c) Natality d) Mortality
Ans. (b)
a) Level PP is “phytoplanktons” in sea and
“Whale” on top level TC
CBSE PRELIMS – 2011
b) Level one PP is “pipal trees” and the
5. Mass of living matter at a trophic level in an level SC is “sheep”
area at any time is called: c) Level PC is “rats” and level SC is “cats”.
a) Standing crop b) Detritus d) Level PC is “insects” and level SC is
c) Humus d) Standing state “small insectivorous birds”
Ans. (a) Ans. (d)
6. Which one of the following statements is 11. Which one of the following is not a functional
correct for secondary succession? unit of an ecosystem
a) It begins on a bare rock a) Decomposition b) Productivity
b) It occurs on a deforested site c) Stratification d) Energy flow
c) It follows primary succession Ans. (c)
58
DR. ARVIND’S BIOLOGY CLASSES
(A Unit of Med-Xel Tutorials)
12. Which one of the following is not a gaseous 14. Which of the following representations shows
biogeochemical cycle in ecosystem? the pyramid of numbers in a forest ecosystem?
a) Phosphorus cycle b) Nitrogen cycle
c) Carbon cycle d) Sulphur cycle
Ans. (a)
Hint: Sulphur cycle has both gaseous and
sedimentary phases.

CBSE MAINS – 2010


a) D b) A c) B d) C
Ans. (d)
13. Study the cycle shown below and select the
option which gives correct words for all the CBSE MAINS – 2011
four blanks A, B, C and D.
15. Which one of the following animals may
occupy more than one trophic levels in the
same ecosystem at the same time?
a) Sparrow b) Lion
c) Goat d) Frog
Ans. (a)
Hint: • Frog and lion are carnivorous
• Goat is herbivorous
• Sparrow is omnivorous.
16. Both, hydrarch and xerarch successions lead
to:
a) Medium water conditions
b) Xeric conditions
c) Highly dry conditions
d) Excessive wet conditions
Ans. (a)
17. The breakdown of detritus into smaller
particles by earthworm is a process called:
a) Humification b) Fragmentation
Options: c) Mineralisation d) Catabolism
A B C D Ans. (b)
a) Nitrification Ammonification Animal Plants
CBSE MAINS – 2012
b) Denitrification Ammonification Plants Animals
18. The second stage of hydrosere is occupied
c) Nitrification Denitrification Animals Plants
by plants like:
d) Denitrification Nitrification Plants Animals a) Salix b) Vallisneria
c) Azolla d) Typha
Ans. (b) Ans. (b)
Hint: The second stage of hydrosere is
occupied by submerged aquatic plants
e.g., Hydrilla, Vallisneria. The third stage
has free floating plants e.g., Azolla
(floating aquatic fern). The fourth stage is
Reed Swamp plants like Typha. Salix
includes deciduous trees and shrubs
which constitute the sixth (woodland
stage) and climax stages.
59
DR. ARVIND’S BIOLOGY CLASSES
(A Unit of Med-Xel Tutorials)
19. Identify the likely organisms (A), (B), (C) and decomposition is largely an oxygen
(D) in the food web shown below: requiring process.
b) False; Leaching is the moving down of water
soluble inorganic nutrients into the soil
horizon.
c) True
d) False; Humus is highly resistant to microbial
action and gets decomposed at a very slow
rate.

AIPMT – 2014
Options:
23. If 20J of energy is trapped at producer level,
(A) (B) (C) (D) then how much energy will be available to
(a) rat dog tortoise crow peacock as food in the following chain?
(b) squirrel cat rat pigeon Plant  mice  snake  peacock
(c) deer rabbit frog rat a) 0.0002J b) 0.02 J
(d) dog squirrel bat deer c) 0.002 J d) 0.2J
Ans. (c) Ans. (b)
20. The rate of formation of new organic matter
by rabbit in a grassland is called: 24. Match the following and select the correct
a) Net primary productivity option:
b) Gross primary productivity (1) Earthworm (i) Pioneer species
c) Net productivity
(2) Succession (ii) Detritivore
d) Secondary productivity
Ans. (d) (3) Ecosystem service (iii) Natality
(4) Population growth (iv) Pollination
(1) (2) (3) (4)
NEET – 2013
a) (ii) (i) (iv) (iii)
21. Secondary productivity is rate of formation of b) (i) (ii) (iii) (iv)
new organic matter by: c) (iv) (i) (iii) (ii)
a) Consumer b) Decomposer
d) (iii) (ii) (iv) (i)
c) Producer d) Parasite
Ans. (a) Ans. (a)
22. Which one of the following processes during 25. Given below is a simplified model of
decomposition is correctly described? phosphorus cycling in a terrestrial ecosystem
a) Catabolism - Last step in the decomposition with four blanks (1 – 4). Identify the blanks
under fully anaerobic condition
b) Leaching – Water soluble inorganic Consumer 3
nutrients rise to the top layers of soil.
c) Fragmentation – Carried out by organism
such as earthworm 4
1
d) Humification – Leads to the accumulation
of a dark coloured substance humus
which undergoes microbial action at a
very fast rate.
Ans. (c) Uptake
Hint: Soil solution
a) False, All the three processes i.e., Fragmentation, Run off
leaching and catabolism operate simultaneously
and is followed by humification and 2
Mineralisation. The process of
60
DR. ARVIND’S BIOLOGY CLASSES
(A Unit of Med-Xel Tutorials)
1 2 3 4 b) cycle sulphur
a) Producers Litter fall Rock Detritus Sedimentary nutrient Nitrogen and
minerals cycle Phosphorus
b) Rock Detritus Litter fall Producers
minerals Gaseous nutrient Nitrogen and
c) Litter fall Producers Rock Detritus c) cycle sulphur
mineral Sedimentary nutrient Carbon and
d) Detritus Rock Producer Litter fall cycle Phosphorus
minerals Gaseous nutrient Sulphur and
Ans. (d)
d) cycle Phosphorus
Sedimentary nutrient Carbon and
AIPMT – 2015 cycle Nitrogen

Ans. (a)
26. Vertical distribution of different species
29. During ecological succession:
occupying different levels in a biotic
a) The gradual and predictable change in
community is known as:
species composition occurs in a given
a) Stratification b) Zonation
area.
c) Pyramid d) Divergence
b) The establishment of a new biotic
Ans.(a)
community is very fast in its primary
27. The mass of living material at a trophic level
phase.
at a particular time is called:
c) The numbers and types of animals
a) Standing state
remain constant.
b) Net primary productivity
d) The changes lead to a community that is
c) Standing crop
in near equilibrium with the environment
d) Gross primary productivity
and is called pioneer community
Ans. (c)
Ans. (a)
Hint:
a) Standing state – The amount of inorganic 30. Most animals that live in deep oceanic
nutrients present any time in soil/water of waters are:
the ecosystem. a) primary consumers
b) Net primary productivity – It is the total b) secondary consumers
organic matter stored by producers per c) tertiary consumers
unit area/volume per unit time. d) detritivores
c) Standing crop – It is the amount of living Ans. (d)
biomass in an ecosystem. 31. In an ecosystem the rate of production of
organic matter during photosynthesis is
d) Gross primary productivity – It is the total
termed as:
organic matter synthesized by the a) Gross primary productivity
producers in the process of b) Secondary productivity
photosynthesis per unit area per unit time. c) Net productivity
28. In which of the following both pairs have d) Net primary productivity
correct combination? Ans. (a)
32. Secondary Succession takes place on/in:
Gaseous nutrient Carbon and
a) Degraded forest
a) cycle Nitrogen b) Newly created pond
Sedimentary nutrient Sulphur and c) Newly cooled lava
cycle Phosphorus d) Bare rock
Gaseous nutrient Carbon and Ans. (a)
61
DR. ARVIND’S BIOLOGY CLASSES
(A Unit of Med-Xel Tutorials)

AIPMT – 2016
33. Which one of the following is a characteristic
feature of cropland ecosystem?
a) Least genetic diversity
b) Absence of weeds
c) Ecological succession
d) Absence of soil organisms
Ans. (a)

34. Which of the following would appear as the


pioneer organisms on bare rocks?
a) Liverworts b) Mosses
c) Green algae d) Lichens
Ans. (d)
35. The term ecosystem was coined by:
a) A.G. Tansley b) E. Haeckel
c) E. Warming d) E.P. Odum
Ans. (a)

NEET – 2017

36. Which ecosystem has the maximum


biomass?
a) Forest ecosystem
b) Grassland ecosystem
c) Pond ecosystem
d) Lake ecosystem
Ans. (a)
37. Presence of plants arranged into well-
defined vertical layers depending on their
height can be seen best in:
a) Tropical Savannah b) Tropical Rain Forest
c) Grassland d) Temperate Forest
Ans. (b)
62
DR. ARVIND’S BIOLOGY CLASSES
(A Unit of Med-Xel Tutorials)
BIODIVERSITY AND CONSERVATION 7. Which one of the following areas in India is
a hotspot of biodiversity?
a) Gangetic Plain b) Sunderbans
CBSE PRELIMS – 2009 c) Western Ghats d) Eastern Ghats
Ans. (c)
1. Tiger is not a resident in which one of the CBSE MAINS – 2010
following national park?
a) Gir b) Jim Corbett 8. The Indian Rhinoceros is a natural
c) Ranthambhor d) Sunderbans inhabitant of which one of the Indian states?
Ans. (a) a) Uttarakhand b) Uttar Pradesh
c) Himachal Pradesh d) Assam
CBSE PRELIMS – 2010 Ans. (d)
2. Which one of the following is an example of CBSE MAINS – 2011
ex-situ conservation?
a) National park b) Wildlife sanctuary 9. Biodiversity of a geographical region
c) Seed bank d) Sacred groves represents:
Ans. (c) a) Endangered species found in the region
b) The diversity in the organisms living in the
region
CBSE PRELIMS – 2011 c) Genetic diversity present in the dominant
3. Which one of the following expanded forms species of the region
of the following acronyms is correct? d) Species endemic to the region
a) IPCC = International Panel for Climate Change Ans. (b)
b) UNEP = United Nations Environmental Policy
c) EPA = Environmental Pollution Agency CBSE MAINS – 2012
d) IUCN = International Union for Conservation
10. Select the correct statement about
of Nature and Natural Resources
biodiversity:
Ans. (d)
a) Western Ghats have a very high degree
Hint: IPCC – Intergovernmental Panel on
climate change. of species richness and endemism
UNEP – United Nations Environmental b) Conservation of biodiversity is just a fad
Programme pursued by the developed countries
EPA – Environment Protection Act c) The desert areas of Rajasthan and Gujarat
4. Which one of the following have the highest have a very high level of desert animal
number of species in nature? species as well as numerous rare animals
a) Fungi b) Insects d) Large scale planting of Bt cotton has no
c) Birds d) Angiosperms adverse effect on biodiversity
Ans. (b) Ans. (a)
5. Large Woody Vines are more commonly 11. Sacred groves are specially useful in:
found in a) Year round flow of water in rivers
a) Temperate forests b) Mangroves b) Conserving rare and threatened species
c) Tropical rainforests d) Alpine forests c) Generating environmental awareness
Ans. (c) d) Preventing soil erosion
Ans. (b)
CBSE PRELIMS – 2012
NEET – 2013
6. The highest number of species in the world
12. Which one of the following is not used for ex
is represented by:
situ plant conservation?
a) Mosses b) Algae
a) Shifting cultivation b) Botanical Gardens
c) Lichens d) Fungi
c) Field gene banks d) Seed banks
Ans. (d)
Ans. (a)
63
DR. ARVIND’S BIOLOGY CLASSES
(A Unit of Med-Xel Tutorials)
AIPMT – 2014 Ans. Grace marks were awarded to all
candidates for this question.
13. The organization which publishes the Red Hint: The given diagram is ambiguous; it
List species is: does not show any significant difference
a) WWF b) ICFRE in the size of the sections B, C, D and thus
c) IUCN d) UNEP it is not possible to assign the category to
Ans. (c) molluscs, crustaceans and other animal
14. A species facing extremely high risk of groups. Thus, any of the two options (a)
extinction in the immediate future is called: or (b) could be correct.
a) Extinct b) Vulnerable 16. An example of ex situ conservation is
c) Endemic d) Critically endangered a) Sacred Grove b) National Park
Ans. (d) c) Seed Bank d) Wildlife Sanctuary
Hint: Ans. (c)
a) Extinct – When taxon has been
completely eliminated from earth. AIPMT – 2015
b) Vulnerable – Presently population is
17. Cryopreservation of gametes of threatened
sufficient but is undergoing depletion so species in viable and fertile condition can be
that it is facing risk of extinction in referred to as:
medium term future. a) Advanced ex-situ conservation of biodiversity
c) Endemic – A species confined to that b) In situ conservation by sacred groves
region and not found anywhere else. c) In situ cryo-conservation of biodiversity
d) In situ conservation of biodiversity
d) Critically endangered – The taxon facing
Ans. (a)
very high risk of extinction in the wild 18. In which of the following both pairs have
and can become extinct any moment in correct combination?
the immediate future. a) In situ conservation: Cryopreservation
15. Given below is the representation of the Ex situ conservation; Wildlife Sanctuary
extent of global diversity of invertebrates. b) In situ conservation: Seed Bank
What groups the four portions (A-D) represent Ex situ conservation: National Park
c) In situ conservation: Tissue culture
respectively?
Ex situ conservation: Sacred groves
d) In situ conservation: National Park
Ex situ conservation: Botanical Garden
Ans. (d)

AIPMT – 2016
19. Which is the National Aquatic Animal of India?
a) River dolphin
b) Blue whale
A B C D c) Sea-horse
a) Insects Molluscs Crustaceans Other d) Gangetic shark
animal Ans. (a)
groups 20. Which of the following is the most important
b) Insects Crustaceans Other animal Molluscs cause of animals and plants being driven to
groups
extinction?
c) Crustaceans Insects Molluscs Other
animal
a) Alien species invasion
groups b) Habitat loss and fragmentation
d) Molluscs Other animal Crustaceans Insects c) Co-extinctions
groups d) Over-exploitation
Ans. (b)
64
DR. ARVIND’S BIOLOGY CLASSES
(A Unit of Med-Xel Tutorials)
NEET-2; 2016 Ans. (a)
28. The region of Biosphere Reserve which is
21. How many hot spots of biodiversity in the legally protected and where no human
world have been identified till date by activity is allowed is known as:
Norman Myers?
a) Core zone b) Buffer zone
a) 17 b) 25 c) 34 d) 43
Ans. (c) c) Transition zone d) Restoration zone
22. Which of the following is correctly matched? Ans. (a)
a) Aerenchyma – Opuntia
b) Age pyramid – Biome
c) Parthenium hysterophorus – Threat to
biodiversity
d) Stratification – Population
Ans. (c)
23. Red list contains data or information on
a) All economically important plants
b) Plants whose products are in international
trade
c) Threatened species
d) Marine vertebrates only
Ans. (c)
24. Which of the following National Parks is
home to the famous musk deer or hangul?
a) Keibul Lamjao National Park, Manipur
b) Bandhavgarh National Park, Madhya
Pradesh
c) Eaglenest Wildlife Sanctuary, Arunachal
Pradesh
d) Dachigam National Park, Jammu and
Kashmir
Ans. (d)

NEET- 2017

26. Alexander Von Humbolt described for the


first time:
a) Ecological Biodiversity
b) Laws of limiting factor
c) Species area relationships
d) Population Growth equation
Ans. (c)
27. Which one of the following is related to Ex-
situ conservation of threatened animals and
plants?
a) Wildlife Safari parks
b) Biodiversity hot spots
c) Amazon rainforest
d) Himalayan region
65
DR. ARVIND’S BIOLOGY CLASSES
(A Unit of Med-Xel Tutorials)
d) Moderately toxic
Ans. (c)
ENVIRONMENTAL ISSUES 6. Global agreement in specific control
stratagies to reduce the release of ozone
CBSE PRELIMS – 2009 depleting substances, was adopted by:
a) The Kyoto Protocol
1. Chipko movement was launched for the b) The Vienna Convention
protection of: c) Rio de Janeiro Conference
a) Livestock b) Wet lands d) The Montreal Protocol
c) Grasslands d) Forests Ans. (d)
Ans. (d) Hint: The Kyoto Protocol was for decreasing
2. Steps taken by the Government of India to greenhouse gas emission.
control air pollution include: Earth Summit (Reo di genaria) was to
a) Permission to use only pure diesel with decrease green house gases.
a maximum of 500 ppm sulphur as fuel 7. Montreal Protocol aims at:
for vehicles. a) Control of water pollution
b) Use of non-polluting Compressed b) Control of CO2 emission
Natural Gas (CNG) only as fuel by all c) Reduction of ozone depleting
buses and trucks. substances
c) Compulsory mixing of 20% ethyl alcohol d) Biodiversity conservation
with petrol & 20% biodiesel with diesel. Ans. (c)
d) Compulsory PUC (Pollution Under
Control) certification of period driven CBSE PRELIMS – 2010
vehicles which tests for carbon
monoxide and hydrocarbons 8. A renewable exhaustible natural resource is:
Ans. (d) a) Forest b) Coal
3. Biochemical Oxygen Demand (BOD) in a c) Petroleum d) Minerals
river water: Ans. (a)
a) Gives a measure of Salmonella in the 9. The two gases making highest relative
water. contribution to the greenhouse gases are:
b) Increases when sewage gets mixed with a) CO2 and N2O b) CO2 & CH4
river water c) CH4 and N2O d) CFC5 & N2O
c) Remains unchanged when algal bloom Ans. (b)
occurs. 10. dB is a standard abbreviation used for the
d) Has no relationship with concentration of quantitative expression of:
oxygen in the water. a) A certain pesticide
Ans. (b) b) The density of bacteria in a medium
4. Which of the following plant species you c) A particular pollutant
would select for the production of bioethanol: d) The dominant Bacillus in a culture
a) Pongamia b) Jatropha Ans. (c)
c) Brassica d) Zea mays
Ans. (d)
Hint: Besides sugarcane and maize, potato, CBSE PRELIMS – 2011
sugarbeet, Tapioca etc. can be used for 11. Eutrophication is often seen in:
production of alcohol. Jatropa is a a) Deserts b) Fresh water lakes
petro plant. c) Ocean d) Mountains
5. DDT residues are rapidly passed through Ans. (b)
food chain causing biomagnification because 12. Which one of the following pairs of gases are
DDT is: the major cause of “Greenhouse effect”?
a) Non-toxic to aquatic animal a) CO2 and O3 b) CO2 and CO
b) Water soluble c) CFCs and SO2 d) CO2 and N2O
c) Lipo soluble Ans. (d)
66
DR. ARVIND’S BIOLOGY CLASSES
(A Unit of Med-Xel Tutorials)
Hint: CO2 accounts for 60%, methane 20%, d) The river water is still suitable for drinking
CFCs 14% and N2O 6%. as impurities are only about 0.1%
13. Which one of the following statements is Ans. (c)
wrong in case of Bhopal tragedy? CBSE MAINS – 2011
a) Methyl Isocyanate gas leakage took place
b) Thousands of human beings died 18. “Good ozone” is found in the
c) Radioactive fall out engulfed Bhopal a) Mesosphere b) Troposphere
d) It took place in the night of December c) Stratosphere d) Ionosphere
2/3, 1984. Ans. (c)
Ans. (c)
CBSE MAINS – 2012
CBSE PRELIMS – 2012
19. The domestic sewage in large cities:
14. In an area where DDT had been used a) When treated in STPs, does not really
extensively, the population of birds declined require the aeration step as the sewage
significantly because: contains adequate oxygen
a) earthworms in the area got eradicated b) Has very high amounts of suspended
b) cobras were feeding exclusively on birds solids and dissolved salts.
c) many of the birds eggs laid, did not hatch c) Has a high BOD as it contains both
d) birds stopped laying eggs aerobic and anaerobic bacteria
Ans. (c) d) Is processed by aerobic and then anaerobic
15. Measuring Biochemical Oxygen Demand bacteria in the secondary treatment in
(BOD) is a method used for: Sewage Treatment plants (STPs)
a) Working out the efficiency of oil driven Ans. (d)
automobile engines Hint:
b) Measuring the activity of Saccharomyces • Domestic sewage contains large amount
cerevisae in producing curd on a of organic matter and microbes.
commercial scale Suspended solids and dissolved salts are
c) Working out the efficiency of R.B.Cs more seen in Industrial waste water.
about their capacity to carry oxygen • STPs require both the aerobic and the
d) Estimating the amount of organic matter anaerobic steps.
in sewage water • When Aerobic bacteria act break down
Ans. (d) the organic matter of sewage they
16. Which one of the following is a wrong statement? consume oxygen. The BOD of the sewage
a) Ozone in upper part of atmosphere is goes on decreasing with the decrease in
harmful to animals organic matter. BOD is a measure of the
b) Greenhouse effect is a natural phenomenon amount of organic matter it does not
c) Eutrophication is a natural phenomenon indicate the presence of bacteria.
in freshwater bodies Thus first three options are incorrect
d) Most of the forests have been lost in
tropical areas NEET – 2013
Ans. (a)
20. Kyoto Protocol was endorsed at:
CBSE MAINS – 2010 a) CoP – 6 b) CoP - 4
c) CoP – 3 d) CoP – 5
17. When domestic sewage mixes with river water Ans. (c)
a) Small animals like rats will die after Hint: Cop stands for conference of Parties
drinking river water which is the governing body of the
b) The increased microbial activity releases convention on Biological Diversity and
micro-nutrients such as iron advances implementation of the convention
c) The increased microbial activity uses up through the decisions it takes at its
dissolved oxygen periodic meetings. Till date it has held 11
major meetings.
67
DR. ARVIND’S BIOLOGY CLASSES
(A Unit of Med-Xel Tutorials)
The kyotoprotocol was adopted at the third d) water is pure
session of the conference of parties in 1997 Ans. (a)
in Kyoto, Japan. 27. The UN Conference of Parties on climate
21. Global warming can be controlled by: change in the year 2011 was held in:
a) Increasing deforestation, slowing down a) South Africa b) Peru
the growth of human population. c) Qatar d) Poland
b) Increasing deforestation, reducing Ans. (a)
efficiency of energy usage. 28. Rachel Carson’s famous book “Silent Spring”
c) Reducing deforestation, cutting down is related to:
use of fossil fuel. a) Noise pollution
d) Reducing reforestation, increasing the b) Population explosion
use of fossil fuel. c) Ecosystem management
Ans. (c) d) Pesticide pollution
22. The Air Prevention and Control of Pollution Ans. (d)
Act came into force in: 29. Which of the following is not one of the prime
a) 1985 b) 1990 health risks associated with greater UV
c) 1975 d) 1981 radiation through the atmosphere due to
Ans. (d) depletion of stratospheric ozone?
a) Reduced Immune System
AIPMT – 2014 b) Damage to eyes
c) Increased liver cancer
23. A location with luxuriant growth of lichens on d) Increased skin cancer
the trees indicates that the Ans. (c)
a) Location is not polluted
b) Trees are very healthy
AIPMT – 2015
c) Trees are heavily infested
d) Location is highly polluted
30. The UN conference of Parties on climate
Ans. (a)
24. The zone of atmosphere in which the ozone change in the year 2012 was held at:
layer is present is called a) Durban b) Doha
a) Troposphere b) Ionosphere c) Lima d) Warsaw
c) Mesosphere d) Stratosphere Ans. (b)
Ans. (d) 31. Increase in concentration of the toxicant at
25. A scrubber in the exhaust of a chemical successive trophic levels is known as:
industrial plant removes:
a) Particulate matter of the size 2.5 micrometer or a) Biomagnification
less b) Biodeterioration
b) Gases like sulphur dioxide c) Biotransformation
c) Particulate matter of the size 5 d) Biogeochemical cycling
micrometer or above Ans. (a)
d) Gases like ozone and methane 32. Acid rain is caused by increase in the
Ans. (b) atmospheric concentration of:
a) SO2 and NO2 b) SO3 and CO
AIPMT – 2015 c) CO2 and CO d) O3 and dust
Ans. (a)
26. High value of BOD (Biochemical Oxygen 33. Eutrophication of water bodies leading to
Demand) indicates that: killing of fishes is mainly due to non-
a) water is highly polluted availability of:
b) water is less polluted a) food b) light
c) consumption of organic matter in the c) essential minerals d) oxygen
water is higher by the microbes Ans. (d)
68
DR. ARVIND’S BIOLOGY CLASSES
(A Unit of Med-Xel Tutorials)
AIPMT – 2016 d) Mortality of fish due to lack of oxygen
Ans. (d)
34. A river with an inflow of domestic sewage 40. The highest DDT concentration in aquatic
rich in organic waste may result in: food chain shall occur in
a) Increased population of aquatic food web a) Phytoplankton b) Seagull
organisms c) Crab d) eel
b) An increased production of fish due to
biodegradable nutrients Ans. (b)
c) Death of fish due to lack of oxygen
d) Drying of the river very soon due to algal
NEET- 2017
bloom
Ans. (c)
35. A system of rotating crops with legume or 41. Which one of the following statements is not
grass pasture to improve soil structure and valid for aerosols?
fertility is called a) They are harmful to human health
a) Contour farming b) They alter rainfall and monsoon patterns
b) Strip farming c) They cause increased agricultural
c) Shifting agriculture
d) Ley farming productivity.
Ans. (d) d) They have negative impact on
36. Depletion of which gas in the atmosphere agricultural land
can lead to an increased incidence of skin Ans. (c)
cancers:
a) Ozone b) Ammonia
c) Methane d) Nitrous oxide
Ans. (a)
37. Joint Forest Management Concept was
introduced in India during:
a) 1970s b) 1980s
c) 1990s d) 1960s
Ans. (b)

NEET-2; 2016
38. Biochemical Oxygen Demand (BOD) may
not be a good index for pollution for water
bodies receiving effluents from
a) Domestic sewage
b) Dairy industry
c) Petroleum industry
d) Sugar industry
Ans. (c)
39. A lake which is rich in organic waste may
result in
a) Increased population of aquatic
organisms due to minerals
b) Drying of the lake due to algal bloom
c) Increased population of fish due to lots of
nutrients

You might also like